[ /tv/ /rf/ /vg/ /a/ /b/ /u/ /bo/ /fur/ /to/ /dt/ /cp/ /oe/ /bg/ /ve/ /r/ /mad/ /d/ /mu/ /cr/ /di/ /sw/ /hr/ /wh/ /lor/ /s/ /hau/ /slow/ /gf/ /vn/ /w/ /ma/ /azu/ /wn/ ] [ Main | Settings | Bookmarks | Music Player ]

No.87526 Reply
File: bourbaki2.jpg
Jpg, 28.41 KB, 194×259 - Click the image to expand
edit Find source with google Find source with iqdb
bourbaki2.jpg
Наша кафедра продолжает свою работу. Здесь мы обсуждаем математику, математиков и математические университеты, а также отвечаем на вопросы доброанонов.

Ссылка на учебники в предыдущем треде.

Предыдущий: >>84296
>> No.87529 Reply
>>87516
= 0, если n = 2m+1
= (-1)^m * C(2m, m), если n = 2m
>> No.87532 Reply
Львовский.-Курс-лекций-по-математическому-анализу.pdf
А вот вам "Анализ" отечественный, баклажанный.
>> No.87534 Reply
>>87521
>>87522
Ну чтобы повторений было.
Вот пример для 0,1,2,3

zero
0
1
2
3
0,1
0,2
0,3
1,2
1,3
2,3
1,2,3
0,2,3
0,1,3
0,1,2
0,1,2,3

А дальше я уже вручную буду выбирать из списка, тут, например, из всех подходит только {0} {1}(т.к. 1 единичный элемент берем допустим) {1,3} {0,1,2,3}
Или есть способ легче найти подгруппы для {0...9}? нежели вариант что нам дали, отсортирововая где нет единичного элемента?
>> No.87538 Reply
>>87534
Количества таких подмножеств заданной длины от 1 до 10 есть 9-я строчка треугольника паскаля:
| | 1 | | 9 | | 36 | | 84 | | 126 | | 126 | | 84 | | 36 | | 9 | | 1 | | 
   Даже если тебе будут их сразу надиктовывать правильно, рука всё равно отвалится.
   Алсо, сгенерить как выше все подмножества для {0...8}, но с пропуском цифры и добавить эту цифру.
Школьник?
>> No.87540 Reply
>>87538
Смотри, {0...9} x+y+3mod10
Задание найти все подгруппы и построить решетку к ним. Вот и сижу мучаюсь.
По идее все подгруппы это те где есть единичный, а также сам элемент и обратный к нему. Но посмотри на количество их 10!
>> No.87550 Reply
>>87540
Общая алгебра какая-то кодировщическая. Не умею в неё пока. Скоро придёт кто-нибудь покруче, может и поможет.
>> No.87551 Reply
матанон реквистируб решебник Алфутовой, гуглил, я верю что у тебя он есть, выручи няш, спасибо
Так же учебников по переодическим последовательностям для начинающих, буду просто безумно благодарен
>> No.87554 Reply
>>87540
Ассоциативность потерял, нет?
>> No.87565 Reply
>>87534
Это же обычные размещения без повторений. (9!/(9-k)!), где "к" -- число от 0 до 9.
>> No.87582 Reply
File: picture-60705.gif
Gif, 497.17 KB, 240×169 - Click the image to expand
edit Find source with google Find source with iqdb
picture-60705.gif
>>87565
Спасибо, мы уже поняли. Их, ясен перец, слишком много для перебора. Надо как-то находить нужные подгруппы, решая уравнения.
>> No.87635 Reply
>>87582
>>87565
>>87554
>>87550
Я разобрался с ней и успешно сдал, спасибо всем.
Вопрос теперь в другом, тот что был про целую и дробную часть. (x-[x]) ну вы помните, так вот. Мощность их -счетное (ну дробных частей), а всего их континуум, как это грамотно записать?
>> No.87640 Reply
>>87635
Наоборот, мощность каждого класса эквивалентности счётная (алеф 0), а мощность фактор-множества - континуум
>> No.87641 Reply
>>87640
т.е. даже просто x у нас счетное, а не только целая часть?([x])
>> No.87649 Reply
>>87641
Честно говоря, не понимаю вопрос. Вот решение:
x и y >= 0 в одном классе эквивалентности, если их дробная часть совпадает.
Каждый класс эквивалентности однозначно характеризуется дробной частью чисел внутри него => классов эквивалентности столько же, сколько различных дробных частей, т.е. чисел на [0, 1) => мощность фактор-множества - континуум

Внутри класса эквивалентности с дробной частью a у нас лежат числа 0+a, 1+a, 2+a, ... => мощность каждого класса эквивалентности счётная
>> No.87663 Reply
Реквестирую картинку с матаном из самого страшного квантмеха.
>> No.87665 Reply
>>87663
> матаном
> квантмеха
Фу, бля.
Всё норм с вами, у меня никаких претензий, но хуй знает почему мне постоянно мерзко от математики. Хотя я подписан на тред и вроде как когда-то некоторую часть математики вполне спокойно воспринимал. Неформальную аксиоматическую теорию множеств изучал, пытался в ней постороить анализ в непрерывном поле.
Ну ладно, продемонстрировал свой БРАУЭР, теперь можно нажимать на кнопку "Отправить".
>> No.87683 Reply
File: 122223s.png
Png, 6.74 KB, 1000×900 - Click the image to expand
edit Find source with google Find source with iqdb
122223s.png
Анон, я ничтожество и ошибся с факультетом, но всё же помоги разобраться с пикрелейтед, завтра контрольная.
>> No.87688 Reply
File: квантмех.jpg
Jpg, 152.71 KB, 568×900 - Click the image to expand
edit Find source with google Find source with iqdb
квантмех.jpg
>>87663
Квантмех - это не самое "страшное". Но, раз уж просишь, вот тебе рандомная страничка из http://c3v.ru/books/math/demidovich_matosnovy_kvantmeh.djvu
>> No.87689 Reply
>>87683
Странное задание. Точно там корень из x?
Что такое "обратимо с какой-нибудь стороны"?
По-моему т.к. f(2-e) = f(2) то оно необратимо
>> No.87690 Reply
File: motivator-очевидно.png
Png, 162.36 KB, 710×576 - Click the image to expand
edit Find source with google Find source with iqdb
motivator-очевидно.png
>> No.87692 Reply
>>87689
> Что такое "обратимо с какой-нибудь стороны"?
Обратимость слева эквивалентна инъективности, справа - сюръективности
g - левая обратная к f если g(f(x)) = x для любого x
g - правая обратная к f если f(g(y)) = y для любого y
Если существует просто обратная, то она является и левой и правой и других ни левых, ни правых нет. А вот когда есть только левая или только правая, то их может быть больше одной.
>> No.87694 Reply
File: 123123123123.png
Png, 4.69 KB, 640×546 - Click the image to expand
edit Find source with google Find source with iqdb
123123123123.png
>>87689
> Точно
Точно.
> "обратимо с какой-нибудь стороны"?
Понимал бы, не пришлось переписывать. По определению как-то так.
>> No.87695 Reply
>>87690
Это действительно очевидно.
>>87683
У вас кообласти, как минимум, нет.
>>87694
Это неправильно определение.
>> No.87696 Reply
>>87695
В правой множество перепутал, да.

>>87694
> Точно
Нет, неточно, из -х, на другой пример посмотрел.
>> No.87697 Reply
>>87695
> Это неправильно определение.
Чем? Путаницей с переменными? Ещё интересно, зачем х сдвоенное.
Вон я написал: >>87692
Но мне лешь доставать тетрадку и возиться. У вас область определения поделена на три множества. Попробуйте найти образы этих множеств. И сделать обратные 3 функции, действующие из этих образов. Если подозреваете фейл, то попробуйте от определения от противного доказать.
>> No.87698 Reply
File: 002624180gbl1.jpg
Jpg, 428.66 KB, 712×1072 - Click the image to expand
edit Find source with google Find source with iqdb
002624180gbl1.jpg
>>87697
Спасибо, сейчас сам повожусь и кину сюда.
>> No.87699 Reply
>>87697
> Путаницей с переменными
Там тождественное отображение не то.
> Но мне лешь доставать тетрадку и возиться.
В уме решилось, если бы не
1. Область прибытия не указана.
2. У логарифма основания нет.
Для множеств существование обратных тождественно сюръективности/инъективности.
>> No.87700 Reply
>>87699
> 2. У логарифма основания нет.
Значит, берём дефолтное - e.
>> No.87701 Reply
>>87700
Для дефолтных специальные записи - ln=loge и lg=log{10}.
Если e, то левого обратного не может быть: образ нат. логарифма от (0;4) пересекается с (1;+\infty).
Про правые ничего сказать невозможно.
>> No.87702 Reply
>>87701
> (1;+\infty).
[1;+\infty).
>> No.87703 Reply
>>87701
Это в России. У буржуев log часто означает ln.
>> No.87704 Reply
>>87701
> невозможно
не возможно
>>87703
Это неправильные буржуи, которые пишут неправильные логарифмы. enwiki://Natural_logarithm - буржепедия в качестве пруфа.
>> No.87705 Reply
>>87704
http://www.wolframalpha.com/input/?i=ln+e%5E2
Вольфрамальфа в качестве пруфа.
>> No.87706 Reply
>>87703
>>87701
>>87700
Нет, это значит что я писал мугичкой.
Повозился, прозреваю фэйл.
>> No.87707 Reply
File: 12222213s.png
Png, 18.73 KB, 1066×1412 - Click the image to expand
edit Find source with google Find source with iqdb
12222213s.png
>> No.87715 Reply
Что за наркоманство?
>>87705
Чому имена функций у языках погроммирования не пишутся латехом.
>>87707
А откуда у вас минус появился под корнем? Его не было.
Сюръективность для записи f(x)=... вообще бессмысленна, потому что не понятно, куда функция бьёт.
> wtf
Если вас беспокоит, что полученные множества из области значений пересекаются, то зря: во-первых, полный прообраз сохраняет объединения, во-вторых, без инъективности у сюрьъективной функции много обратных, т.е. на пересекающейся области нужно выбрать какие-то из возможных значений произвольно.
>> No.87719 Reply
>>87715
> Чому имена функций у языках погроммирования не пишутся латехом.
Чому там log, а не ln?
>> No.87722 Reply
>>87719
Рекурсивная дискуссия.
>> No.87829 Reply
>>87526
Анон, кто мне может дать линк на пошаговое объяснение вычисления ротора вект поля? Я не пойму как эта матрица составляется. Вот пример a=yzi-xzj+xyk
Г: x^2+y^2+z^2=9
   x^2+y^2=9

Вот где тут что при составлении? Что тут i,j,k, ax, ay az и d/dx и т.д. при составлении? В интернете всюду готовый ответ выводят, а мне надо именно узнать как составлять для теоремы стокса.
Заранее спасибо.
>> No.87834 Reply
>>87829
ротор посчитал, вышел 2xi+2yj-2zk верно? Как тогда дальше применять формулу учитывая нормаль и Г
>> No.87856 Reply
File: TWBAR.jpg
Jpg, 578.79 KB, 2592×1944 - Click the image to expand
edit Find source with google Find source with iqdb
TWBAR.jpg
Доброанон, как посчитать lim x-->1 (e^(x)-e)/ln x без тейлора и лопиталя?
>> No.87863 Reply
>>87856
Хм. Чем-то на второй замечательный похоже.
>> No.87867 Reply
>>87856
= e lim y->0 (e^y - 1) / ln(1 + y) = e lim y->0 (e^y - 1) / y lim y->0 y / ln(1 + y)

lim y->0 ln(1 + y) / y = ln[ lim y->0 (1 + y)^(1/y) ] = ln e = 1
lim y->0 (e^y - 1) / y = { z = e^y - 1 => y = ln(1 + z) } = lim z->0 z / ln(1 + z) = 1

В итоге всё стремится к e
>> No.87868 Reply
>>87867
Да благословит тебя Аллах и приветствует.
>> No.87869 Reply
>>87867
бтв, все решалось уже после замены на y:
e lim y->0 (e^y - 1) / ln(1 + y)

e^y-1 ~ y, и ln(1+y) ~ y. Так что получалось e*1=e

Еще раз спасибо!
>> No.87871 Reply
>>87869
а, нет
>> No.87872 Reply
File: CodeCogsEqn.jpg
Jpg, 3.75 KB, 194×57 - Click the image to expand
edit Find source with google Find source with iqdb
CodeCogsEqn.jpg
ох, пиздец я туплю
все ок там после первого щага по эквивалентам

а вот еще одна штука.
дошел до того как использую первый замечательный предел, а дальше не могу понять что делать
>> No.87888 Reply
>>87872
tg3x по эквивалентности на 3x. Дальше у тебя будет 1/x^2 перел скобкой. Одну штуку 1/х занести под знак корня. Пос ел этого неопределённости нет.
>> No.87897 Reply
File: 31739.jpg
Jpg, 39.70 KB, 500×509 - Click the image to expand
edit Find source with google Find source with iqdb
31739.jpg
Суп, няши.
Посоветуйте пособие для подготовки к ЕГЭ по математике?
>> No.87910 Reply
>>87897
reshuege.ru
Ткачук Математика абитуриенту скачать
Киселёв геометрия скачать
Шень Алгебра скачать
Курант Роббинс Что такое математика скачать
Сканави сборник задач для поступающих скачать
Громов гиперболические группы скачать

Петросяннабигал
>> No.87911 Reply
File: 3102860_large.jpg
Jpg, 195.66 KB, 1000×667 - Click the image to expand
edit Find source with google Find source with iqdb
3102860_large.jpg
>> No.87938 Reply
File: 650116.jpg
Jpg, 97.24 KB, 550×550 - Click the image to expand
edit Find source with google Find source with iqdb
650116.jpg
Помоги мне добрый аноним. Дело такое, я вообще не знаю математику даже школьный курс, остановился на 5-7 классе. Мне 23 года, позорище и сейчас я часто сталкиваюсь с криптографией и программированием. Мне нужна математика. Подскажи аноним какие книги читать? Учебники тех классов где остановился или есть еще учебники, что-бы с нуля? И подскажи в каком направлении двигаться.
>> No.87940 Reply
>>87938
> Учебники тех классов где остановился
Школьные учебники - бесполезный хлам. Всё их содержание сводится к "мудрость предков гласит, что вычислять 2+3 нужно так. Блаблабла. Вычислите 3+2". Читать школьные учебники бессмысленно.
Искренне рекомендую тебе вот эту годноту:
http://yadi.sk/d/vdrKZE3-0W0Xy
http://yadi.sk/d/dPI2g4Ct0xFMk
http://yadi.sk/d/YnrtkGlA1X7ER
http://yadi.sk/d/XiEbEEMc1X7XU

Кроме того, скачай книги из предыдущего треда: те, которые в шапке, плюс Рудин.
>> No.87941 Reply
>>87940
Да, вот ещё годный учебник геометрии: http://yadi.sk/d/M9N5-Nnb1X7j0
>> No.87942 Reply
>>87940
М.Рид "Алгебраическая геометрия для няшек"
:3
Только надо товарищу подсказать, что начинать надо с Алексеева и Гельфанд-Шеня, а то он может несколько потерять уверенность в себе, lol.
>> No.87975 Reply
>>87941 >>87942 >>87940 Спасибо огромное, я вас никогда не забуду.
>> No.88055 Reply
С Новым годом, катедра.
>> No.88057 Reply
>>88055
О ДА С НОВЫМ ГОДОСМ !! ПУСТЬ ВАШИ ИНТЕГРАЛЫ БУДУТ ЭЛЕМЕНТАРНЫМИ, ПРОСТРАНСТВА НЕЕВКЛИДОВЫМИ, А ВСЕ РЯДЫ - СХОДЯЩИМИСЯ !!!
>> No.88058 Reply
>>87938
Если есть деньги около 3к за курс, то советую заочную школу МИФИ http://www.mifi.ru. Это лучше самообразования, так как тебя проверит посторонний человек и укажет на твои ошибки.
>> No.88067 Reply
А как здесь относятся к гильбертизму?
>> No.88068 Reply
>>88067
Гильбертизмом в детстве мальчики занимаются, а здесь кафедра математики.
>> No.88069 Reply
>>88068
> математики
> Гильбертизм
А где противоречие?
>> No.88076 Reply
>>88067
Что такое гильбертизм?
>> No.88084 Reply
>>88076
Символизм, формализм Бурбаки-стайл. Всякий математический объект тождественен символу, которым он обозначается. Математика - раздел лингвистики.
>> No.88177 Reply
В десятичной системе дробь 1\3 записывается как 0;333... А как
эта же дробь запишется в двоичной системе? Помогите
>> No.88186 Reply
Доброанончики, объясните на пальцах варианты нахождения матричной экспоненты. Пропустил в своё время возведение матриц в различные степени, да и в целом с линейной алгеброй знаком лишь поверхностно потому и нужна помощь. Пытаясь нагуглить материал попадаюсь на решение дифуров с помощью неё, в котором экспоненте почти не уделяют внимания, подробно расписывая решение уравнений. Честно гугил учебники, но пролистывая оглавление ничего похожего не заметил. С меня обещание поднатаскать знание линала в ближайший год.

tl;dr
Дана экспонента, как возвести её в степень матрицы?
>> No.88188 Reply
>>88177
Прозреваю 0,(010) впрочем, считал в уме, так что если что, меня поправят.
>> No.88193 Reply
>>88186
Матричная экспонента определяется с помощью ряда Тейлора.
Напомню, разложение экспоненты в ряд Тейлора выглядит так: e^x = 1 + x + (x^2)/2! + (x^3)/3! + ... Пусть есть матрица M, которую можно умножать саму на себя неограниченное число раз. Тогда exp M есть
e^M = sum(k=0; inf)(M^k)/k!, где M^k есть M*M*{k раз}*M.
>> No.88195 Reply
>>88193
> есть M*M*{k раз}*M
M - квадратная матрица размерности r
M^0 = E (единичная матрица размерности r, то есть квадратная матрица, на главной диагонали которой стоят единицы)
M^1 = M
M^2 = M x M
M^3 = M x M x M
и т.д.

В общем,
M^0 = E
M^k = M^(k-1) * M, k >0, k натуральное.
>> No.88199 Reply
>>88193
Няша, сказанное тобой вполне ясно, но как мне быть с конкретным примером? Допустим дана матрица А, неужто мне придется считать сумму ряда от матрицы с факториалом? Я же в "боевых" условиях этого не сделаю. Да я и от простого то ряда вряд ли сумму посчитаю, если это не 1/n или 1/n^2, которые всем известны.
>> No.88204 Reply
>> No.88205 Reply
Копперсмит и Виноград
>> No.88207 Reply
File: Матрицы.png
Png, 180.60 KB, 1280×1024 - Click the image to expand
edit Find source with google Find source with iqdb
Матрицы.png
Этот тред захватывают Копперсмит и Виноград матрицы, всюду они.
>> No.88229 Reply
>> No.88266 Reply
Анон, я правильно понимаю, что если k - ранг матрицы, то в качестве строчечного/столбцового базиса можно взять любые k линейно независимых строк/столбцов?
>> No.88269 Reply
>>88266
Верно. Это, в общем-то, из определения ранга следует.
>> No.88270 Reply
>>88204
Ты СОВЕРШЕННО не понимаешь в чем суть математики. Математика это не численные эксперименты в стиле «чуваки, щас я нажму на кнопку и оно выдаст мне гигабайт бреда». Математика это не псевдоинтеллектуальные пляски вокруг искусственного псевдоинтеллекта. Математика это не эмуляция real life'a, не разведение бактерий в фарфоровой чашке и даже не статистические исследования смертности от рака в зависимости от количества съеденных чипсов. Математика это место, где люди могут погрузиться в астрал — бесконечный, глубокий и абсолютно точно отражающий нашу реальность.
Физики строят двадцать красивых теорий - мы считаем когомологии и обрушиваем их все без всякого коллайдера. Единая Россия побеждает на выборах, а мы раскладываем количество участков по процентам явки и видим пики на процентах, делящихся на пять.
Генетики не могут решить комбинаторную задачу, а мы находим интересные связи с комбинаторикой пространств модулей, ничуть не помогая им. Количественные методы, исследования устойчивости, случайные процессы — а мы рассматриваем бесконечномерную алгебру Ли и делаем далеко идущие выводы. Динамика численности популяции кроликов в Южном Уэльсе — мы рассматриваем бесконечномерную алгебру Ли. Сверхпроводники при комнатной температуре - мы снова зачем-то рассматриваем бесконечномерную алгебру Ли и просим ещё.
Мы бездушно включим в нашу геометрию любую неудавшуюся теорию всего, наши предпочтения не основаны на здравом смысле, абстрактные обобщения — наша стихия, мы — истинное лицо логики этого мира.
>> No.88271 Reply
>>88270
Прикладно-математик-кун смеётся и плачет с этих слов. LOL :(
>> No.88287 Reply
>>88271
Это уже интересно.
Ты настоящий "прикладной" математик? Всегда хотел встретить такого живьём. Нет, ну, микроорганизмов, которые занимаются моделированием какой-нибудь игрушечно-экономической хуеты я видел немало, а вот чтобы человек глубоко знал численные методы, алгоритмы, всякие хитровыебанные оценки сложности, такого нет (честно говоря, особо и не искал, просто любопытно).

Ну рассказывай, что знаешь, какие интересы. Что из математики тебе полезно/интересно?
>> No.88290 Reply
>>88287
Ты настоящий "прикладной" математик?

Да, всаммамделишный. Решаю задачи типа, "вот тут у нас такая техническая проблема, чё нам делать?", до моделировании игрушечной хуеты, которая возможно через несколько десятков лет будет использоваться инженерами. А может и небудет, это часто в фундаментальных исследованиях.

Мои интересы: математическое моделирование. Есть немного инженерных знаний, если надо пишу симуляции, делаю статистический анализ, разрабатываю алгоритмы.
>> No.88294 Reply
>>88290
Не корми их, у них тут скорее лингвистика, без какого либо приложения для конкретных целей.
>> No.88298 Reply
>>88290
То, что ты описал в первом абзаце, в чистом виде инженерная деятельность. Я не сомневаюсь, что моделирование чего-то полезного в народном хозяйстве задача совсем непростая, но математика там тривиальная, не представляющая для науки интереса.

Но это в любом случае любопытно. Напиши, например, какие учебники по этой деятельности ты бы посоветовал.
>> No.88299 Reply
>>88294

Спасибо за предупреждение. Но иногда некоторые мои коллеги (к счастью мало) действительно ерундой страдают штампуя публикации которые и с математической, и с практической точки зрения имеют сомнительную ценность.
>> No.88304 Reply
>>88294
> у них тут скорее лингвистика
Будто что-то плохое. Изучение математики невозможно без знания языка математики.
> без какого либо приложения для конкретных целей
Похоже, ещё один нигилист. Ты-то сам хотя бы "Тривиум" Арнольда осилишь?
>> No.88305 Reply
>>88298
> То, что ты описал в первом абзаце, в чистом виде инженерная деятельность. Я не сомневаюсь, что моделирование чего-то полезного в народном хозяйстве задача совсем непростая, но математика там тривиальная, не представляющая для науки интереса.
Математическое моделирование, как не странно, требует знание математики. И действительно решение задачи может быть просто. Найти простое решение, для не простой проблемы, далеко не просто. Когда это происходит я этому очень рад. И что-бы понять техническую сторону вопроса я использую свое инженерное образование. Но когда алгоритмов нет, я разрабатываю собственные а так-же анализирую системы для которых нету ещё нет решений.

Если ты думаешь, что математика ценна, только года в ней могут разобраться 3.5 человека, это не так.
> Но это в любом случае любопытно. Напиши, например, какие учебники по этой деятельности ты бы посоветовал.
Читать например журналы из области "Operations Research".
>> No.88306 Reply
>>88290
Няша, няша! Я давно тебя ждал. Можешь насоветовать годной литературы, желательно на русском языке? Хочу разобраться в математике, нужной для понимания Ландау-Лифшица. Очень буду тебе признателен.
>> No.88307 Reply
>>88306
> Няша, няша! Я давно тебя ждал. Можешь насоветовать годной литературы, желательно на русском языке? Хочу разобраться в математике, нужной для понимания Ландау-Лифшица. Очень буду тебе признателен.
Рад бы, но я получил своё образование в Германии. Я математические выражения перевожу на русский, при помощи википедии.

Ты сначала скажи, какая прикладная сторона вопроса тебя интересует. Или математическое моделирование как таковое?
>> No.88309 Reply
>>88287
> Ну рассказывай, что знаешь, какие интересы. Что из математики тебе полезно/интересно?
Что мне полезно: теория графов, статистика, вычислительная сложность, линейная алгебра, случайный процессы, эвристические методы, аппроксимация, вычислительная математика.

В данный момент только интересно: топология, дифференциальное уравнение простые и в частных производных, теория игр, динамические системы, конечные элементы.
>> No.88314 Reply
>>88307
> я получил своё образование в Германии
Вызывает антирес ваш технический прогресс: как у вас там сеют брюкву - с кожурою али без?

Покажи, будь ласка, учебники (на немецком), чтобы можно было оценить уровень и сравнить с моим универчиком.
>> No.88320 Reply
>>88314
Я думаю, это можно самостоятельно отыскать.
>> No.88321 Reply
>>88314

первые 2 года:

линейная алгебра: Lineare Algebra: Eine Einführung für Studienanfänger, Gerd Fischer.

Математический анализ: Analysis 1,2 Konrad Konigsberger; Analysis 1,2,3 Otto Forster

потом
теория графов: Reinhard Diestel, Graphentheorie (есть на английском).

дифференциальное уравнение простые и в частных производных: Partial Differential Equations (Graduate Studies in Mathematics) Lawrence C. Evans. (английский).


Остальное конспекты лекции (lecture notes) профессоров. Я думаю книги недостаточны для показателя уровня университета. Важно у какого професора посещать лекции.
>> No.88324 Reply
>>88270
Понимаю, что спрашивать у пасты глупо, но какой матаппарат используется при описании сверхпроводников?
>> No.88326 Reply
>>88321
Двадцать евро за книжку? Ну и цены.
> Я думаю книги недостаточны для показателя уровня университета.
Превьюшек хватило, чтобы погрустнеть.
>> No.88327 Reply
>>88326
Насчет двадцати евро. У меня иногда такое чувство, что в Германии высшее техническое образование очень закрыто от населения, которое не имеет к нему отношения. Если в наших книжных магазинах, можно все-таки найти кое-какие книги по математике, физике, химии, причем довольно сложные и хорошие, а также книги на английском, то в Германии такое сейчас только заказать можно. Ну и вообще, деление по мозгам там начинается еще с малых лет, когда по основным, реальным школам и гимназиям распределяют. И закон об авторском праве там очень работает, так что немецких книжек в инернете тоже очень сложно найти. Ну и сравнить: живут там больше 90 миллионов человек, а высшее образование полчает всего около 2(!) миллионов, вроде бы. Может у меня ложное впечатление сложилось, но по-моему для немцев вышка - это действительно что-то элитарное, и не нужное большинству населения, меньше проблем.
>> No.88338 Reply
>>88327
> Если в наших книжных магазинах, можно все-таки найти кое-какие книги по математике
Это где? В РФ?
Если в РФ, то тут за исключением нескольких (одного-двух) магазинов да и то с очень бедным ассортиментом не продаются никакие математические книги вообще. То есть совсем, буквально. Потому что российские издательства такого уже давно не выпускают, а английские книжки не завозят.
Подозреваю, что с другими науками дело обстоит так же.
>> No.88341 Reply
>>88338
> тут за исключением нескольких (одного-двух) магазинов да и то с очень бедным ассортиментом не продаются никакие математические книги вообще
Томск-кун с тобой не согласится.
>> No.88360 Reply
>>88327
> в Германии высшее техническое образование очень закрыто от населения, которое не имеет к нему отношения...
> Ну и вообще, деление по мозгам там начинается еще с малых лет, когда по основным, реальным школам и гимназиям распределяют.
да :( , дети рабочих имеют в среднем 2.6 раз меньше шансов попасть в гимназию. Шанс получить высшее образование без законченной гимназии очень низкий.

http://www.sueddeutsche.de/karriere/uebertritt-aufs-gymnasium-mit-leis[...]206-2
> то в Германии такое сейчас только заказать можно.
Не думаю что это большая проблема, есть библиотеки.
> Ну и сравнить: живут там больше 90 миллионов человек, а высшее образование полчает всего около 2(!) миллионов, вроде бы.
Нет, примерно каждый десятый.

http://www.welt.de/wams_print/article1547533/Nur-jeder-zehnte-Deutsche[...].html
> И закон об авторском праве там очень работает, так что немецких книжек в инернете тоже очень сложно найти.
Вряд ли из-за строгих законов. В США закон строже. Книг больше. Просто мало людей читают немецкую литературу.
> Может у меня ложное впечатление сложилось, но по-моему для немцев вышка - это действительно что-то элитарное, и не нужное большинству населения, меньше проблем.
Я полагаю не только "не нужное" но и "не доступное" большенству. Пример со школой смотри выше.
>> No.88362 Reply
>>88326
> Двадцать евро за книжку? Ну и цены.
Это очень дешёвые книги. (Не потому что цена маленькая, а потому что дешевле других книг).
> > Я думаю книги недостаточны для показателя уровня университета.
> Превьюшек хватило, чтобы погрустнеть.
Почему?

Вот тебе ещё книги по основам (Grundlagen) механики (1-курс) и электротехники:
Grundlagen der Technischen Mechanik, Kurt Magnus, Hans H Müller. Moeller Grundlagen der Elektrotechnik (49 ЕВР, за новую, б/у смотри на amazon.de)
>> No.88364 Reply
>>88338
Продаются почти в каждом магазине у нас учебники по анализу, линейной алгебре, теории вероятности, дискретной математике, к примеру. Да - бедно, да - только на русском, но кое-что есть. В Германии я специально заходил в несколько магазинов, ничего сложнее популярных книжек типа Гарднера и Хокинга не видел.
>> No.88365 Reply
Это при том, что книг по гуманитарным дисциплинам (толстенных книг по истории, политике, социологии, философии) - хоть отбавляй. Это заговор.
>> No.88366 Reply
>>88365
Скорее, закон спроса и предложения в действии.
>> No.88367 Reply
>>88364
Анализ и линейная алгебра (как учебные курсы) это, конечно, основы, но по объёму от всего, что нужно выучить математику из математики (независимо от специализации) занимают процента три. Книжек, например, по гомологической алгебре или дифференциальной топологии в продаже найти невозможно.
>> No.88368 Reply
>>88364
> В Германии я специально заходил в несколько магазинов
Смею предположить, что в Германии гораздо больше магазинов, чем ты посетил. Но даже если ты где-то далеко на периферии, можно в онлайн магазине заказать.
>> No.88369 Reply
>>88367
> занимают процента три.
Будьте любезны, докажите это утверждение.
> Книжек, например, по гомологической алгебре или дифференциальной топологии в продаже найти невозможно.
Логично. В России мало специалистов, которым интересна дифференциальная топология. Однако такого рода книжки можно заказать в научных библиотеках или в книжных магазинах ВУЗов и купить без наценки (или с небольшой наценкой; во всяком случае, дешевле, чем на амазоне). Это, конечно, не открытая продажа, но и не полное отсутствие продаж.
>> No.88370 Reply
>>88367

Для математика не обязательно знать гомологическую алгебру или дифференциальную топологию. Это уже специализация.
>> No.88372 Reply
File: Contents.pdf
Pdf, 0.20 KB, 419×595 - Click the image to get file
Contents.pdf
>>88362
> Почему?
Вот содержание полугодового курса "Линейная алгебра и аналитическая геометрия - 1", который читается в моём универе. Найдите десять отличий.
>> No.88373 Reply
>>88369
> которым интересна дифференциальная топология
дифференциальная топология входит в число азбучных дисциплин для математиков и физиков-теоретиков, есличо
(то что математиков и теорфизиков в России мало это да, факт)
> полное отсутствие продаж
Именно полное. В библиотеках найти можно. Но некоторые книжки мне бы, например, хотелось иметь дома постоянно.
>> No.88374 Reply
>>88373
> Но некоторые книжки мне бы, например, хотелось иметь дома постоянно.
Ну так за чем дело стало? Во многих библиотеках барыжат книгами по цене издательства.
> дифференциальная топология входит в число азбучных дисциплин для математиков и физиков-теоретиков, есличо
В курсе. А когда ты последний раз с живым теорфизиком встречался?
>> No.88375 Reply
>>88327
А библиотеки?
>>88365
Гуманитарные дисциплины нужны всем: все голосуют за политиков, общаются с людьми, многие ищут заказы и имеют подчинённых, судятся, картины на выставках смотрят, читают худлит ну и конечно мелют языком про любовниц короля какого-нибудь. Технические/естественные/математика - только на производстве, инженерам, кодерам, учёным и т.п. В жизни достаточно школьного уровня для общего развития. Proove me wrong.
>>88366
Ну конечно же.
>>88374
> Ну так за чем дело стало? Во многих библиотеках барыжат книгами по цене издательства.
Можно ксерокс сделать, только это не очень дёшево, хотя наверное как и сама книга.
>> No.88380 Reply
>>88372
> > Почему?
> Вот содержание полугодового курса "Линейная алгебра и аналитическая геометрия - 1", который читается в моём универе. Найдите десять отличий.
Посмотрел, это же для информатиков. У нас для информатиков тоже более простая математика. Так-что вроде ничего страшного.
>> No.88384 Reply
>>88375
> Гуманитарные дисциплины нужны всем ...
О каком уровне гуманитарных дисциплин здесь идет речь? Начальная школа или высшее образование?
> Технические/естественные/математика ...
Аналогичный вопрос, о каком уровне идёт речь?
>> No.88386 Reply
>>88380
Но не настолько же. Полгода рассказывать про умножение матриц друг на друга, да так и не рассказать - это издевательство.
>> No.88388 Reply
>>88386
> Но не настолько же. Полгода рассказывать про умножение матриц друг на друга, да так и не рассказать - это издевательство.
Лично для меня эти темы и кажутся простыми, но я всё же не знаю в каких условиях проводятся эти курсы, какая их цель и какая их целевая аудитория. Я время от времени преподаю информатикам, и среди них есть достаточно талантливых или трудолюбивых учеников, способных работать с математикой на уровне математиков.

Так ты информатик?
>> No.88390 Reply
>>88388
Да, информатик.
> какая их цель
Официально заявленная - позволить понимать и делать вот такое, например: http://habrahabr.ru/post/148901/ . На практике матрица определяется как таблица чисел (не уточняется, каких), словосочетания "матрица над кольцом/телом/полем" в курсе не встречаются, про связь матриц с операторами упоминается вскользь (и, естественно, не объясняется, почему умножение матриц определяется именно таким, не слишком очевидным образом).
> какая их целевая аудитория
Суровые буряты, которые первые два месяца пытаются осознать, что A+B не всегда равно B+A. Они любят шутить про Нео, а вопрос "что такое кольцо" считают завальным.
>> No.88391 Reply
>>88384
О любом. Об уровне чуть выше школьного на 3/4 в простой школе. О чём ты вообще сказать хотел? Не знал, что никому в основном не интересны планеты и частицы, а интересная реальная жизнь? Не говоря уже о сферах и бубликах, которые вообще только в головах математиках существуют.
>> No.88395 Reply
>>88324
Поднимаю вопрос.
>> No.88396 Reply
>>88324
Зависит от уровня сложности работы. Сходи на архив.орг и посмотри.
>> No.88397 Reply
>> No.88402 Reply
>>88397
Интересно, кто этим универам логотипы рисует? Небузь кучу денег отвалили дорогому дизайнеру за крутую элитную байду, красивый шрифт и цвет и толщину полосочк. Помню, в нашем лицее устроили конкурс на рисование логотипа с гербом. Лучше это не видеть. Так они ещё хотели это к пиджакам на рукав прилепить, как военным. Так и так, понимаешь, одна из 10-ти лучших школ России.
>> No.88406 Reply
>>88391
> > 88375
Ты сформулировал своё мнение настолько невнятно и противоречиво, что мне требуется больше информации для того, что бы ходя бы понять основную мысль и аргументацию в её пользу.

> О любом. Об уровне чуть выше школьного на 3/4 в простой школе
Выражения "О любом." и "Об уровне чуть выше школьного" противоречат друг другу.

"выше школьного на 3/4". Числовое измерение уровня образование мне не известно.

Предположим ты имеешь ввиду "уровень чуть выше школьного". Я правильно понимаю ниже перечисленные твои утверждение?:

1) Всем людям, нужно общаться с людьми, искать заказы и иметь подчинённых, судится, и смотрит картины. Для этого им (людям) нужно уровень гуманитарного образования чуть выше школьного.

2) В отличии от гуманитарных наук, технические, естественные науки и математика уровнем чуть выше школьного нужны только на производстве, инженерам, программистам и учёным.

3) В жизни всем людям достаточно школьного уровня для общего развития.
>> No.88408 Reply
>>88390
> Официально заявленная - позволить понимать и делать вот такое, например: http://habrahabr.ru/post/148901/ .
Интересная статья. Спасибо.
> > какая их цель
> На практике матрица определяется как таблица чисел (не уточняется, каких), словосочетания "матрица над кольцом/телом/полем" в курсе не встречаются, про связь матриц с операторами упоминается вскользь (и, естественно, не объясняется, почему умножение матриц определяется именно таким, не слишком очевидным образом).
Понимаю твою критику и с ней согласен.
> > какая их целевая аудитория
> Суровые буряты, которые первые два месяца пытаются осознать, что A+B не всегда равно B+A. Они любят шутить про Нео, а вопрос "что такое кольцо" считают завальным.
Пичаль. У тебя есть возможность посещать лекции математиков?
>> No.88409 Reply
>>88408
> У тебя есть возможность посещать лекции математиков?
Нет. На местных мехматах ситуация, в общем, такая же, а до Москвы три с половиной тысячи километров. Занимаюсь самообразованием, сейчас вот трактат Бурбаки грызу.
Ладно, заканчиваю пятиминутку нытья, извините за внимание.
>> No.88414 Reply
>>88409
> Занимаюсь самообразованием, сейчас вот трактат Бурбаки грызу.
Респект. Нытья не заметил.

Почитай ещё истрорию Бурбаки. История математики может быть довольно интересна.
>> No.88417 Reply
>>88406
> 1) Всем людям, нужно общаться с людьми, искать заказы и иметь подчинённых, судится, и смотрит картины. Для этого им (людям) нужно уровень гуманитарного образования чуть выше школьного.
Именно.
> 2) В отличии от гуманитарных наук, технические, естественные науки и математика уровнем чуть выше школьного нужны только на производстве, инженерам, программистам и учёным.
Да.
> 3) В жизни всем людям достаточно школьного уровня для общего развития.
Тут речь о технических, естественных и математике.
>> No.88418 Reply
>>88417
> > 1) Всем людям, нужно общаться с людьми, искать заказы и иметь подчинённых, судится, и смотрит картины. Для этого им (людям) нужно уровень гуманитарного образования чуть выше школьного.
> Именно.
Пардон, тут я имел в виду, что чем выше тем лучше, а не чуть выше.
>> No.88419 Reply
В который раз невежды путают гуманитарные науки с культурой, это становится утомительно.
Смотреть картины, читать книжки, иметь интересных собеседников -- это культура. Социология, политология, искусствоведение -- это гуманитарные "науки". Вы чувствуете разницу, невежда?
>> No.88420 Reply
>>88419
Ну так я говорю, что из-за смотрения на картины и ради смотрения, они изучают искусствоведение, а чтобы иметь собеседников - социологию с психологией, а чтобы голосовать - политологию, историю. Вот почему эти науки популярнее математики с механикой среди непрофессионалов.
>> No.88421 Reply
>>88420
Забавно, многие технари, особенно на бордах, известны тем, что часто отрицают и гуманитарные науки и едвали не саму культуру из-за того, что их критерии качества и доказательства теорий не устраивают гносеологически. Мол, нельзя доказать существование подсознания и крутизну Герники также точно, как физики доказывают закон тяготения, значит, что Герника и подсознание - это ложь, а художники и психотерапевты - шарлотаны, а рисовать как Пикассо могут все.
>> No.88422 Reply
>>88420
Я надеюсь на самом деле я даже не надеюсь вы понимаете, что вы чрезвычайно тупой? (но я допускаю что вы просто крайне неумело выражаете свои мысли).
> чтобы иметь собеседников - социологию с психологией
> ради смотрения, они изучают искусствоведение
Нет, ну и дурак, мне даже неловко.
>> No.88423 Reply
>>88422
Ты мне тоже нравишься. Я не настаиваю на том, что они правы, делая это. Я лишь говорю, что они это делают. Вот почему тех книг больше? А что не так, бля? Ты бы хоть объяснил, раз надеешься.
>> No.88424 Reply
>>88423
Нет, ну перечитай это >>88420, там каждое утверждение феерическая глупость. Мне кажется, ты хотел сказать что-то другое.
Никто не читает учебник социологии, чтобы иметь собеседников, я не знаю что тут разжёвывать.
Ты когда-нибудь видел человека, читавшего учебник политологии, чтобы проголосовать на выборах?

>>88421
> гуманитарные науки и едвали не саму культуру
Это не связанные друг с другом вещи. Что значит "отрицать кудьтуру" мне неведомо (контркультура?), а вот что гуманитарные "науки" науками не являются это довольно очевидно. Потому что наука это когда можно более-менее объективно отличить правду от вранья. Очевидно, в политологии и пр. этого сделать нельзя.
> подсознание
Ну и во-вторых. Мелкий штришок. Если бы вы читали что-то из психотерапии, то знали бы, что в русском (и английском и, насколько я помню, немецком) языке общепринятым среди психологов/психотерапевтов является термин бессознательное. Вы бы ознакомились с терминологией перед тем как выступать.
>> No.88425 Reply
>>88424
> Никто не читает учебник социологии, чтобы иметь собеседников, я не знаю что тут разжёвывать.
> Ты когда-нибудь видел человека, читавшего учебник политологии, чтобы проголосовать на выборах?
Видел кучу и тех и других людей, людей интересующихся техническими науками, которые бы не использованли их учёбе или работе, не видел.
> Ну и во-вторых. Мелкий штришок. Если бы вы читали что-то из психотерапии, то знали бы, что в русском (и английском и, насколько я помню, немецком) языке общепринятым среди психологов/психотерапевтов является термин бессознательное. Вы бы ознакомились с терминологией перед тем как выступать.
Верное замечание, протупил.
> контркультура?
Нет, её отсутствие, отричание всего вокруг и уподобление Диогену, живущему в бочке, только без нравственности.
>> No.88426 Reply
>>88424
> Никто не читает учебник социологии, чтобы иметь собеседников
Могу привести много контрпримеров.
>> No.88427 Reply
>>88426
Т.е. сидит битард дома и думает "не умею я общаться с людьми, почитаю что ли учебник социологии, чтобы были общие темы", так?
это Кащенко выходит на связь?

А почему не учебник ихтиологии, например?
>> No.88428 Reply
>>88427
Да, именно так. И психологию за этим же читают.
Ичсх, добиваются успехов. "Социализация" называется.
>> No.88429 Reply
>>88427
Именно, но не для тем, а думает, что поймёт общество и станет выше в нём, сможет с людьми общаться, применит социлогию и психологию на практике, вот сколько этих "тренингов" понаделали, бабки наворачиваю на битардах нормальные.
>> No.88430 Reply
>>88417

Ты попробуй не принимать радикальных позиций вроде "гуманитарные науки против математики". Ведь таже самая математика, может например помогать в общении. Я предполагаю, по предыдушим постам, для тебя это важно. Поэтому я тебе советую почитать о такой области математики как "логика первого порядка" и "теории множеств" на начальном уровне. Возможно это тебе поможет яснее выражаться. Например рассмотрим твои утверждения:

2) В отличии от гуманитарных наук, технические, естественные науки и математика уровнем чуть выше школьного нужны только на производстве, инженерам, программистам и учёным.

3) В жизни всем людям достаточно школьного уровня технических, естественных и математике для общего развития. (принял к сведению исправление 88417)

2) и 3) противоречат друг другу.

Посмотри на этот маленький пример, и просто подумай как ты можешь его исправить.

Если можно, можешь пожалуйста сказать какое у тебя образовании или интересы? Мне было бы легче аргументировать, если бы я знал какие знания у моего собеседника.
>> No.88431 Reply
>>88427
Алсо, забавно, но даже если читать ради тем, и то от социологии больше толку чем от ихтиологии. Рыбы не интересны никому вообще, за разговоры про рыб тебя вообще посчитают сумасшедшим. Зато вот ихтиологии всё доказано хорошо, не покритикуешь особо.
>> No.88432 Reply
>>88429
наворачиваюТ
Фикс.
>> No.88435 Reply
>>88430
> таже самая математика, может например помогать в общении. Я предполагаю, по предыдушим постам, для тебя это важно. Поэтому я тебе советую почитать о такой области математики как "логика первого порядка" и "теории множеств" на начальном уровне. Возможно это тебе поможет яснее выражаться.
Не поможет, люди ценят только когда выражаешься красиво, а не точно. Если разговаривать как автор учебника по теории множеств, тебя тоже сумасшедшим посчитают.
> 2) и 3) противоречат друг другу.
Имеется в виду, в жизни для тех, кто не пользуется на работе. И вообще в жизни вне работы. Когда ничё не разрабатываешь, не чинишь и не строишь.
> Если можно, можешь пожалуйста сказать какое у тебя образовании или интересы?
Отделение "прикладной математики и информатики", кодер. За такие же попытки прояснять слова собеседника по логике, на меня часто косо смотрят.
>> No.88437 Reply
( это >>88427 я забыл неймфагнуть )
>>88428
>>88429
Оставляя в стороне научность этих дисциплин, и психология, и социология имеют вполне конкретный специализированный объект изучения. Прочитав учебник социологии можно узнать 27 причин мексиканской революции, но какая ещё "социализация"? Психология это не Дейл Карнеги, лол.
(Почитав кое-что по психотерапии, действительно можно облегчить себе жизнь, но это чтение непростое и уж никак не распространённое в массах).
Социология и психология имеют такое же отношение к жизненной мудрости, как гипотеза Пуанкаре и теорема Гёделя, или как экономика к умению зарабатывать деньги.
>> No.88439 Reply
>>88437
> Социология и психология имеют такое же отношение к жизненной мудрости
Гиппенрейтер в учебнике для первокуров по психологии настойчиво вдалбливает прямо обратное.
> (Почитав кое-что по психотерапии, действительно можно облегчить себе жизнь, но это чтение непростое и уж никак не распространённое в массах).
Распостранено, читают и Фромма, и Берна и доходят до Яломов с Франклами.
> экономика к умению зарабатывать деньги
Её если читают непрофессионалы, то тоже, чтобы строить свои политические убеждения.
>> No.88440 Reply
Я хотел написать длинный текст о методологии науки, но уже запутался, кто в этом треде какие точки зрения отстаивает.
Если кто-то хочет продолжить спор, сформулируйте ваши тезисы, пожалуйста, чтобы я мог их опровергнуть.
>> No.88441 Reply
>>88440
Изначально был поставлен вопрос, почему на полках магазинов книг по гуманитарным дисцеплинам намного больше чем по другим.
>> No.88443 Reply
>>88441
> книг по гуманитарным дисцеплинам намного больше чем по другим
Я бы поставил под сомнение это утверждение, учитывая количество книжек по yoba-программированию, синтаксису разных языков, администрированию, схемотехнике, по калькулусу и "аналитической геометрии" и т.д.

Изначально был вопрос, почему в Москве невозможно купить нормальные базовые учебники по математике, хотя спрос есть. Если бы сейчас издали Хартсхорна, ну 1000 экземпляров, например, снесли бы с полок мгновенно.
>> No.88448 Reply
File: Параноик.png
Png, 104.45 KB, 509×393 - Click the image to expand
edit Find source with google Find source with iqdb
Параноик.png
Анон, после того как я сдам доп. сессию если сдам, дебил блять то за какой учебник из >>84296 мне лучше взяться, чтобы методично расставить всё по полочкам и познать Дзен?
>> No.88449 Reply
Создателю следующего треда надо в ОП-посте написать, чтобы приходящие за советами подробно писали, что именно они хотят изучить, с какой целью, что уже известно, что не получается.
>> No.88453 Reply
>>88448
Демидович - ревизия полученных в российском универе знаний. Учебник Ван дер Вардена - введение в классическую алгебру. Книга Ленга - в каком-то смысле дополнение к Вардену, в этом учебнике уделено внимание тем вещам, которые Варден не осветил. Книга Зорича - лучший из отечественных учебников анализа. Книга Шварца - лучший из всех западных учебников анализа, которые когда-либо переводились на русский язык. Учебники Зорича и Шварца дополняют друг друга, хотя и во многом перекрываются. В качестве пособия по анализу (но не в качестве учебника) посоветую Рудина.

То есть сперва читаешь Демидовича, затем одновременно читаешь Вардена-Ленга и Зорича-Шварца. В процессе сверяешься с Рудиным. Затем читаешь книжки Шабата, "Введение в комплексный анализ", и Мак Лейна, "Категории для работающего математика".
Затем учишь английский, лол.

>>88449
Хорошо, напишу.
>> No.88472 Reply
File: balance-3.jpg
Jpg, 26.01 KB, 600×449 - Click the image to expand
edit Find source with google Find source with iqdb
balance-3.jpg
>>88435
> Не поможет, люди ценят только когда выражаешься красиво, а не точно
Точность никогда не бывает лишней(с). Она важна и в суде, и в математике и в философском споре.
> Если разговаривать как автор учебника по теории множеств, тебя тоже сумасшедшим посчитают.
Разные учебники написаны разным языком.
> И вообще в жизни вне работы.
Возможно применять свои технические знания в обычной жизни. Это называется техническая смекалка. Применение подобных хитростей позволяет существенно сэкономить силы.
Что касается технической литературы, то скажу, что написание учебника по техническому предмету занимает много времени. Это нетривиальный труд, но при этом написать книгу, которая будет полезна большому числу инженеров чрезвычайно сложно.
>> No.88479 Reply
>>88472
> Она важна и в суде, и в математике и в философском споре
Но не за кружкой пива.
>> No.88484 Reply
>>88479
Философские споры как раз за кружкой пива и ведутся. Декарта спроси.
>> No.88485 Reply
>>88435
> > Если можно, можешь пожалуйста сказать какое у тебя образовании или интересы?
> Отделение "прикладной математики и информатики", кодер. За такие же попытки прояснять слова собеседника по логике, на меня часто косо смотрят.
Ну раз техническое образование не смогло привить тебе важность технического образования также для неинжинеров, непрограммистов, неучёных, и не занятых на производстве, мне это вряд ли удастся.
> За такие же попытки прояснять слова собеседника по логике, на меня часто косо смотрят.
Я бы тоже на тебя косо смотрел. Выражаешься ты довольно запутанно. Как в языковом так и в логическом отношении.

Я вот даже точно не знаю что точно обозначает выражение "прояснять слова собеседника по логике".

В любом случае, я с тобой согласен, что гуманитарные науки важны.
>> No.88486 Reply
>>88435
> Если разговаривать как автор учебника по теории множеств, тебя тоже сумасшедшим посчитают.
В подтверждение анонимного коллеги >> 88472

[сарказм] Зато если будешь разговаривать как Шекспир, на староанглийском и в стихотворной форме, никто в твоей вменяемости сомневаться не будет.[/сарказм]

Даже если ты выступаешь на технической конференции, тебе желательно знать, кто твои слушатели, и соответственно подогнать речь.
>> No.88488 Reply
Анон, как доказать, что столбцовый ранг матрицы совпадает со строчным?
Я не знаю, как доказатьстоль очевидный факт, что количество базисных столбцов совпадает с таковым базисных строк.
>> No.88489 Reply
>> No.88490 Reply
File: 1320359302627.jpg
Jpg, 42.55 KB, 356×437 - Click the image to expand
edit Find source with google Find source with iqdb
1320359302627.jpg
>>88489
Премного благодарен.
>> No.88491 Reply
>>88485
> "прояснять слова собеседника по логике"
Указывать ему на логические неточности и просить прояснить их.
>> No.88496 Reply
Рассмотрим функционалы на C[0,1]: I(f) - интеграл f, delta_x(f) = f(x), x \in [0,1].
Тогда I лежит в замыкании линейной оболочки {delta_x, x \in [0,1]} в слабой* топологии. Это очевидно следует из определения интеграла Римана.
На самом деле, любой функционал лежит в этом замыкании. Но это уже не так очевидно.
Может быть интеграл Римана все-таки нужен? Не такой уж он и сложный.
>> No.88496 Reply
Рассмотрим функционалы на C[0,1]: I(f) - интеграл f, delta_x(f) = f(x), x \in [0,1].
Тогда I лежит в замыкании линейной оболочки {delta_x, x \in [0,1]} в слабой* топологии. Это очевидно следует из определения интеграла Римана.
На самом деле, любой функционал лежит в этом замыкании. Но это уже не так очевидно.
Может быть интеграл Римана все-таки нужен? Не такой уж он и сложный.
>> No.88499 Reply
Анончики, а можете рассказать, как можно найти интегрирующий множитель для сведения дифференциальных уравнений к полному дифференциалу? Считал раньше, что решаю всё на нормальном уровне, а тут встретился с таким примером и растерялся. Есть ли алгоритм поиска этой функции, или находится она "наугад", а скилл прокачивается только после долгой практики?
В моей книжке написано лишь, что есть такая функция, а на конкретном примере сказано "Найдем решение следующего уравнения. Интегрирующий множитель будет иметь вид: ..."
>> No.88514 Reply
>>88496
Дело в том, что интеграл Римана не определяется как "нечто лежащее в замыкании delta_x".
Плох он по двум причинам:
(это уже обсуждалось в предыдущих тредах)
1. В определении используется мера Жордана, но студентам это не сообщается.
2. Технически интеграл Римана не проще интеграла Лебега, тогда зачем тратить время на всё равно не нужную ерунду.
>> No.88522 Reply
>>88514
Я имела ввиду одномерный, в нем мера не нужна. Бессмысленность многомерного очевидна, он действительно не проще.
Но одномерный, в отличие от Лебега, можно рассказать за одну лекцию в первом семестре.
Ты сейчас бородат?
>> No.88522 Reply
>>88514
Я имела ввиду одномерный, в нем мера не нужна. Бессмысленность многомерного очевидна, он действительно не проще.
Но одномерный, в отличие от Лебега, можно рассказать за одну лекцию в первом семестре.
Ты сейчас бородат?
>> No.88535 Reply
File: 250px-Hopf_algebra.svg.png
Png, 7.54 KB, 250×224 - Click the image to expand
edit Find source with google Find source with iqdb
250px-Hopf_algebra.svg.png
>>88522
Как это не нужна? Как тогда определить, по каким множествам можно интегрировать?
пиканрилейтед
больше няшек-неймфагов в тред!
>> No.88538 Reply
>>88522
> одномерный, в нем мера не нужна
В одномерном нужна одна мера.
>> No.88555 Reply
File: задать-вопрос.jpg
Jpg, 19.44 KB, 356×316 - Click the image to expand
edit Find source with google Find source with iqdb
задать-вопрос.jpg
>>87526
Привет, анон. Пишет тебе гуманитарий, который сегодня очень пожалел, что не учил математику в школе.
Не мог бы ты помочь решить мне одну задачу, если она решаема? С меня то, что попросишь.
Пикрелейтед. Я знаю положение точек а и b на оси координат и длинну радиуса. Можно ли по этим данным найти положение точки z, да и вообще любой точки на отрезке аb, если да, то отошли, что читать, а если нет, то каких данных не хватает?
>> No.88560 Reply
>>88555
Тебе не приходило в голову, что окружность нарисованная вокруг твоего отрезка вообще какбы ни при чём?
Уравние точек на отрезке с координатами концов (ax;ay) и (bx;by) выглядит так:
x=ax(1-t) + bxt
y=ay(1-t) + byt
0<t<1
Каждому t из интервала соответствует точка на отрезка с координатами из уравнения. При t=0 достигается точка a, при t=1 достигается b, при t=0.5 - середина отрезка, t=1/3 - точка удалённая от a на одну треть отрезка и т.д
>> No.88561 Reply
>>88560
Дурацкий парсер, bx и by это одна переменная, а t другая, там умножение, ну ты понял.
>> No.88640 Reply
Тем временем в /b/ появился интересный тред:
>>/b/2030751
>> No.88682 Reply
>>88640
В подтверждение твоего поста, осмелюсь спросить тут, ибо объяснить мне некому.
При подготовке к экзамену по матану, читая конспект, я наткнулся на то, что не понял еще в начале года, но как-то забил.

Определение предела функции по Коши.
1)Что такое дельта и эпсилон? Нам выдали это как данность, на википедиях это тоже как данность.
2) lx-al<δ lf(x)-Al<ε
Когда я получаю, например, lx-7l<δ lx-7l<ε/3 ЧТО ЭТО ДОКАЗЫВАЕТ?

Объясните, пожалуйста. Я честно хочу понять это для себя.
>> No.88688 Reply
>>88682
Подумав, пришел к выводам:
1)
ε -приращение/уращение по у
δ - приращение/уращение по х
Верно?
2)
   lx-7l<δ lx-7l<ε/3 позволяет записать формулу вида ε/3=δ? Если это так, то как это доказывает существование предела в точке 7?
>> No.88689 Reply
>>88682
эти бесноватые иероглифы означают, что для любой окрестности точки А, большой или маленькой (нас интересуют маленькие, конечно, по большей части) существует проколотая окрестность точки "а", которая полностью туда отобразиться. Куда при этом отобразиться сама точка "а" неважно в случае предела.

Окрестностью точки называется любое открытое множество, содержащее точку.
Проколотой окрестностью, соответственно, окрестность без самой точки.
В случае действительной прямой достаточно из всех открытых множеств рассматривать только интервалы (т.к. они образуют базу топологии), откуда и появляются епсилоны и дельты.
>> No.88690 Reply
>>88682
Эпсилон - произвольно выбранное число. Дельта - функция от эпсилон.

Пусть есть f(x). На OX выбрана точка a, на OY - точка A.
Если для всякого интервала на OY, содержащего точку A, можем предъявить содержащий точку a интервал на OX такой, что все значения функции на этом интервале лежат в выбранном интервале на OY, то "A есть предел f при аргументе, стремящемся к a".

Выбор интервала на OY делается с помощью задания e, соответствующий интервал на OX нам доставляет функция дельта.

Образно говоря, с помощью эпсилон мы указываем отрезочек на OY, а дельта нам доставляет отрезочек на OX. Если докажем, что есть такая функция дельта, то докажем существование предела. Обычно существование дельты доказывается с помощью явного её построения.


Строгое определение на языке топологии есть вот тут: >>87532
>> No.88693 Reply
>>88682
>>88690
Исторически понятие "предел" означало, что мы можем указать такой отрезок на OX, что значения функции на нём сколь угодно мало отличаются от значения функции в некоей определённой точке.
>> No.88695 Reply
>>88682
В нормальных определениях дельта и эпсилон имеют строго заданный смысл.
Какое определение тебя смущат? Выкладывай.
>>88688
Нет, определение у них другое.
>> No.88698 Reply
File: -.jpg
Jpg, 46.89 KB, 639×418 - Click the image to expand
edit Find source with google Find source with iqdb
-.jpg
Сап, няки. Давайте устроим workshop по алгебрам.

Начнём с совсем простых вещей и доберёмся до заоблачных высот до, ну, для начала алгебр Вейля.
>> No.88700 Reply
>>88698
Пусть M - множество, а f - бинарная операция M x M -> M.
Тогда <M, f> - магма :3
>> No.88703 Reply
>>88698
Что такое workshop?
>> No.88721 Reply
>>88703
Мастерская, например.
>> No.88723 Reply
>>88690
Спасибо большое, теперь я понял. :3
А относительно второго вопроса
Значит надо всё-таки получить что-то типа этого ε/3=δ? Ведь есть функция дельта, доставляющая нам эпсилон.
>> No.88725 Reply
>>88721
В данном контексте.
>> No.88823 Reply
>>88723
Нет-нет-нет, эпсилон - это параметр дельты: δ = δ(ε)
> ε/3=δ
x/3 = y, если в школьных обозначениях.
>> No.88824 Reply
File: 86bd3a5628dd8c5d6b835e3f90a1cc1d.png
Png, 491.25 KB, 777×777 - Click the image to expand
edit Find source with google Find source with iqdb
86bd3a5628dd8c5d6b835e3f90a1cc1d.png
Зачем нужны комплексные числа?
Можно ли использовать комплексные числа в ньютоновской механике, и если да, то как?
Какие задачи выгоднее решать в комплексных числах, чем в действительных?
>> No.88826 Reply
>>88823
Фишка в том, что эпсилон (то есть аргумент функции, x) задаёт отрезок на оси OY, а дельта (значение функции, y) доставляет отрезок на оси OX.
>> No.88832 Reply
>>88824
У многочлена степени N действительных корней бывает не более N, а так может быть любое количество, а комплексных ровно N(но некоторые кратные). + Теория вычетов.
>> No.88839 Reply
Аноны, позабыл всю математику. Как можно быстро посчитать вот такое уравнение:
(1+х)^49=1,803
Суть в огромной степени, которую просто так не разложишь. Как вытащить х?
>> No.88843 Reply
>>88839
Эмм... Никак? Разве что автоматически. Откуда ты его взял?
>> No.88846 Reply
>>88824
Аэро- и гидродинамика. ТФКП для этого, собственно, и создавалась.
>> No.88848 Reply
>>88839
> Как можно быстро посчитать вот такое уравнение
> (1+х)^49=1,803
В зависимости от постановки вопроса, ответ:
1) Калькулятором.
2) х=(1,803)^(-49)-1
3) Никак.
>> No.88850 Reply
>>88843
Не ну конечно x = корень 49 степени из 1.803 -1, но оно иррациональное и счить его только итерационными методами можно. Я это имел в виду.
>> No.88852 Reply
>>88843
Накосячил, поэтому результат странный.
исходное вот уравнение
-84,42=21,745/(1+х)^4+21,745/(1+x)^5+...+21,745/(1+x)^10
Судорожно вспоминая что с ним можно сделать, решил привести к общему знаменателю перемножив знаменатели между собой. Основа одинаковая, значит степени складываются, от 4 до 10, итого 49. Но из головы вылетело что ещё надо было домножать числитель. Позор.
А если домножить то через несколько шагов получу
-84,42=21,745(1+х)^6+21,745(1+x)^5+...+21,745 и все это деленное на (1+x)^10.
Дальше видимо придется раскрывать скобки, потому как числитель я больше не знаю как упростить.
>> No.88854 Reply
>>88852
Может лучше юзать мапл или что-нибудь ещё неручное?
>> No.88856 Reply
File: 4ed187f529e529cec7e4e01732d1877d.png
Png, 450.08 KB, 590×708 - Click the image to expand
edit Find source with google Find source with iqdb
4ed187f529e529cec7e4e01732d1877d.png
>>88846
Мне бы конкретики.
>> No.88858 Reply
>>88856
В механике много возникает диференциальных и других уравнений, а в них ТФКП очень много используется. Объясняется это прежде всего тем, что я сказал в этом посте - >>88832. Гугли, например, решение линейных диференциальных уравнений с постоянными коэффицентами. Там алгоритм есть.

Ещё используют при взятии интегралов теорию вычетов. Заключается она, если коротко, в следующем: если функция как-то хорошо определена(диферецируема бесконечнно много раз вроде) в области(на открытом связном множестве), то интеграл по любой кривой в этой области от этой функции равен нулю! А если в области есть несколько особых точек, то он выражается через так называемые "вычеты", посчитанные в этих точках.

ТФКП даёт инструмены определения, где сходится ряд Тейлора различных функций.

Благодаря формуле Эйлера, ТФКП помогает переходить от тригонометрических функций к экспонентам, с которыми легче работать.
>> No.88859 Reply
>>88375
> Гуманитарные дисциплины
> <...>
> для общего развития.
Всё правильно сказал
>> No.88860 Reply
>>88856
> Мне бы конкретики.
Что значит конкретнее? Зачем? Ты реферат что ли пишешь, "Роль функции комплексного переменного в народном хозяйстве"?

С помощью них исследуются вопросы плоского обтекания, например, обтекание воздухом самолетного крыла, для этого есть очень известная функция Жуковского.

В случае плоских течений, при условии потенциальности скорости, вводится комплексный потенциал - он будет аналитической ФКП. Его производной будет вектор скорости. Что мы получили? Мы получили, что с помощью функции комплексной переменной можем выяснить вектор скорости воздуха/воды В ЛЮБОЙ ТОЧКЕ на плоскости (читай в плоском сечении), если на ней есть какой-то твердый объект - хоть круг (круглая опора моста), хоть квадрат (квадратная опора моста), хоть крыло (крыло), хоть полуплоскость (стена здания). А из этого уже сможем определить силы, действующие на объект - нагрузки, подъемную силу и т.п.

Вся магия заключается в том, что нормальная производная вышеупомянутого комплексного потенциала на границе области (объекта) равна 0, а на бесконечности задаем из здравого смысла. После этого ищем эту самую функцию.

Вот тебе более конкретно. Все думал, пригодится ли мне в жизни механика сплошных сред? Пригодилась.
>> No.88861 Reply
>>88858
Это, конечно, всё здорово, но у меня ещё два вопроса есть: >>88824
> Можно ли использовать комплексные числа в ньютоновской механике, и если да, то как?
> Какие задачи выгоднее решать в комплексных числах, чем в действительных?
Если можно, с конкретным примером.
>> No.88864 Reply
>>88861
Вон я те накатал >>88860

Конкретный пример: круглая опора моста. Она же окружность (на плоскости). Комплексный потенциал: W(z)=V(z+(R^2)/z). V - константа, скорость на бесконечности (т.е. считай скорость течения реки).

Скорость воды в точке z - W'(z). Если нарисуешь на бумажке, увидишь линии тока, плавно огибающие окружность, как тебе и подсказывает здравый смысл (зато теперь ты точно знаешь траектории). Ну а в случае с крылом самолета, здравый смысл может забарахлить, а метод - вот он, можешь все подсчитать, выяснить давление сверху и снизу крыла и узнать подъемную силу.
>> No.88870 Reply
>>88861
Прандтль "Гидроаэромеханика", раздел про механику идеальной жидкости.
Фейнман "Фейнмановские лекции по физике", книга вторая, раздел о колебаниях механических систем.
Посмотри короче там, разделы на память сказал, но в этих книгах точно видел применение комплексных чисел.
>> No.88884 Reply
File: STiDhzi8qSc.jpg
Jpg, 194.71 KB, 1280×853 - Click the image to expand
edit Find source with google Find source with iqdb
STiDhzi8qSc.jpg
Воркщоп временно откладывается, начинайте без меня. Для начала я собирался разобрать базовые структурные теоремы для простых/полупростых/коммутативных алгебр.

>>88824
А почему тебя заинтересовала именно ньютоновская механика? Сами по себе комплексные числа тебе не очень нравятся? А кватернионцы?

Комплексные числа описывают повороты плоскости, а кватернионы -- повороты трёхмерного пространства (вокруг разных осей). Но каждому повороту соответствует два кватерниона, причём композиция поворотов соответствует умножению кватернионов.
По-научному говоря, нормированные кватернионы задают двулистное накрытие трёхмерной сферы на SO(3)
>> No.88885 Reply
>>88864
> Если нарисуешь на бумажке, увидишь линии тока, плавно огибающие окружность, как тебе и подсказывает здравый смысл (зато теперь ты точно знаешь траектории).
Чтение учебников по физике всегда у меня вызывали многие тонны негодования фразами подобного сорта. Может физика тоже не наука, как и гуманитарщина? Ну чё это тако?
мимо-математик, не умеющий в физику никак вообще
>> No.88887 Reply
>>88884
> А почему тебя заинтересовала именно ньютоновская механика? Сами по себе комплексные числа тебе не очень нравятся? А кватернионцы?
В этом нет ничего удивительного, комплексные числа сами по себе отдельно от математиков не существуют, а вот механику уже применять можно. У меня например такая классная эспрессо-варка стоит!
>> No.88890 Reply
>>88887
> комплексные числа сами по себе отдельно от математиков не существуют
Это утверждение аналогично тому, что музыкальные произведения не "существуют" для людей, лишенных слуха.
>> No.88891 Reply
>>88890
Комплексное число не имеет эстетической ценности по крайней мере для большинства.
>> No.88893 Reply
>>88891
При чём тут эстетическая ценность?
"Не существует" и "не могу потрогать/услышать/увидеть" вещи между собой никак не связанные.
>> No.88895 Reply
Накидайте материала по С*-алгебрам, епта!
>> No.88897 Reply
>>88893
Математика без ее приложений - множество бесполезных фактов.
>> No.88899 Reply
File: 133745627554851s.png
Png, 136.88 KB, 200×150 - Click the image to expand
edit Find source with google Find source with iqdb
133745627554851s.png
>>88897
С твоим мнением я спорить не буду, на то оно и мнение.
Моё же заключается в том, что приложения не являются целью науки (не только математики, а какой бы то ни было), а наука сама по себе, т.е. изучение того, что вообще поддаётся изучению, и является первоочередной задачей.
>> No.88900 Reply
>>88893
Музыка, когда её играют, явно существует в виде колебаний воздуха. Колебания воздуха реальны и существуют вне зависимости от того, слышат их или нет.
>> No.88914 Reply
>>88900
Теорема, когда её доказывают, явно существует в виде символов на бумаге или мыслей доказывающего. Символы на бумаге реальны и существуют вне зависимости от того, понимают их или нет.
>> No.88923 Reply
>>88914
Все такие умные, а показать зачем нужны комплексные числа не можете.
Рисовать какието линии, какието мосты. Где связь с комплексными числами? Вы вообщетмаьематикой пользоваться умеете? как программисты, которые понтуются терминами, а программ писать не умеют. Что, кроме рисованиядвулепестковой фигни в вакууме примеров искользования комплексных чисел привести не можете?
мимо
>> No.88924 Reply
>>88923
> мимо
Проходим, не задерживаемся.
>> No.88925 Reply
File: 006.png
Png, 2.25 KB, 314×70
edit Find source with google Find source with iqdb
006.png
File: 45720-020530-8ec8...
Jpg, 58.88 KB, 430×475
edit Find source with google Find source with iqdb
45720-020530-8ec8ec6fa8c20c2d40a879e268573ced.jpg

>> No.88926 Reply
>>88925
Определитель матрицы, да. Ну ладно, что это n! слогаемых, каждое из которых произведение элементов матриц я понял. Но как понять, какие это элементы? Вернее что за jn(k).
>> No.88927 Reply
Вот я могу показать маьэтематику в реальной жизни. Например наступил на лё д, он прогибается, я наблюдаю колокол гаусса и знаю когда провалюсь- немного не доходя до дельиафункции. Знаю как быстро размешать сахар в чае. А выслова знаете, а математику не понимаете.
Что, слабо показать комплчисла в реальной жизни?
>> No.88929 Reply
>>88890
> Это утверждение аналогично тому, что музыкальные произведения не "существуют" для людей, лишенных слуха.
А разве не так? Картины не существуют для людей, лишенных зрения, например.
>> No.88931 Reply
File: varg_vikernes.jpg
Jpg, 10.84 KB, 320×350 - Click the image to expand
edit Find source with google Find source with iqdb
varg_vikernes.jpg
>>88929
Они не могут потрогать масло?
>> No.88932 Reply
File: 6189321.jpg
Jpg, 132.71 KB, 963×675
Your censorship settings forbid this file.
unrated
File: 6189321.jpg
Jpg, 132.71 KB, 963×675 - Click the image to expand
edit Find source with google Find source with iqdb
6189321.jpg
Доброанон, разреши узнать у тебя, за что ты любишь математику? В чём твой интерес к ней? Как ты понял, что она тебе по нраву?
>> No.88933 Reply
>>88932
Это как хорошая жвачка. Читанул анализа с топологией, подоказывал всяких вещей по метрическим пространствам, поглядел на алгебры и закончил аналитическим и геометрическим подходами к простым вещам.
Привыкается, расслабляет, хорошо помогает отвлечься.
теорфизик, запрашивал материала по Банаховым пространствам
>> No.88934 Reply
>>88899
Твоё мнение конечно на существование право имеет, но ты же сейчас мнение о ТФКП у неофита опустишь навзнечь. Ну просит он применений, жалко, что ли?
>>88923
Ну он предлагает давление воды на мост мерить, как я понимаю. И то, как энергия воды меняется, возможно. Я сам специализируюсь на алгоритмах, графах и всяком таком и пока мало продвинулся при этом, а в физике и механике слабоват. Я хорошо знаю важность ТФКП в решении дифур и взятии интегралов, но не могу привести примеры задач, где такие интегралы и дифуры возникают.

Алло, механики, тут у кого-нибудь есть примеры задач, приводящих к линейным дифурам с постоянными или к интегралам, берущимся через вычеты?
>> No.88935 Reply
File: Снимок.PNG
Png, 34.90 KB, 547×148
edit Find source with google Find source with iqdb
Снимок.PNG
File: 1311158975684.jpg
Jpg, 37.73 KB, 251×272
edit Find source with google Find source with iqdb
1311158975684.jpg

Halp, все никак не получается.
>> No.88956 Reply
>>88934
Кстати, кто-то в /b в обсуждении универов ор поднял, что без хорошего ТФКП не создать фотошоп. Куда эти умники пропали, а? А?!
>> No.88965 Reply
>>87526
Ребят, есть 35 вопросов по дифференциальным уравнениям. Экзамен через 4 дня, проблема в том, что я вообще не понимаю и не знаю предмет, как только открываю лекции, я не понимаю что там и о чем они, как это можно эффективно решить?
>> No.88969 Reply
>>88932
Ненавижу математику.
>> No.88972 Reply
Ну что же вы, Бетмены?
http://yadi.sk/d/qlZ53Py30balg - "Реальные применения мнимых чисел". Вычисление площади многоугольника, etc.
http://yadi.sk/d/HTroicjV0uCZV - "Комплексные числа и их применение в геометрии".

>>88956
Не сделать. Преобразование Фурье такое комплексное.
ruwiki://Дискретное_преобразование_Фурье
>> No.88976 Reply
>>88927
Арнольд детектед.

Уравнение теплопроводности
Провалился под лёд я без лыж в первые дни мая, переходя по льду входящее теперь в черту Москвы стометровое озеро «Миру — мир». Началось с того, что лёд подо мной стал слегка прогибаться, и под кедами показалась вода. Вскоре я понял, что форма льда — гауссовская колоколообразная (перевёрнутая) кривая. Ещё через минуту стало ясно, что я наблюдаю фундаментальное решение уравнения теплопроводности (в обратном времени). И, действительно, слегка не дойдя до дельта-функции, лёд провалился, и я оказался в проруби диаметром в полметра, метрах в тридцати от берега.
Хотя толщина льда, почти уже белого от многочисленных заполненных водой трещинок, была 5–10 сантиметров, вылезти на него оказалось очень трудно. Этот лёд, который минуту назад удерживал меня вместе с гауссовой лужей метра четыре в диаметре и полметра глубиной в центре, теперь немедленно ломался, когда я клал руку на край проруби. В конце концов прорубь расширилась и позволила мне разогнаться вплавь и выброситься не ещё нетронутый лёд, по которому я и двигался дальше ползком, пока не перешёл озеро до конца.
Связь прогиба льда с уравнением теплопроводности меня уже тогда не удивила, так как я читал воспоминания фон Кармана, который ответил в Москве какому-то специалисту на вопрос о поведении решений одного уравнения с частными производными:
— Это вы хотите танки через Байкал перегонять?
Фон Карман напугал тогда принимавших его лиц своим мгновенным предсказанием серии дождей во Внукове:
— Дело в том, — сказал он, — что над нами сейчас проходит дорожка Кармана.

http://ega-math.narod.ru/Arnold3.htm#ch15
>> No.88978 Reply
>>88935
Ты хочешь, чтобы тебе объяснили, что такое криволинейный интеграл, няша?
>> No.88981 Reply
>>88972
Ценность твоего поста такая же, как и предыдущих. Просили применение в этой самой вашей пошлой реальности, например в механике, а не в другой математике. Много знать, но не мочь решить поставленную задачу не характеризует нас хорошо.
>> No.88983 Reply
>>88981
> Просили применение в этой самой вашей пошлой реальности
Чем тебе вычисление площади куска обоев не реальность?
>> No.88989 Reply
>>88983
Никогда не видел непрямоугольные обои.
>> No.88990 Reply
>>88989
Совсем не знаешь ты жизни, дядя Фёдор.
>> No.88992 Reply
>>88990
Суровые анонимные математики обклеивают стены листями Мёбиуса, а цветы ставят в в вазу Клейна?
>> No.88997 Reply
>>88981
> Просили применение в этой самой вашей пошлой реальности
В этой нашей пошлой реальности не применяется ни одна из наук. Ни математика, ни физика, ни химия, ни философия. Науки - удел погрузившихся в астрал. Обывателям не нужны никакие научные знания, им вполне хватает горстки фактов. 2+2=4. Почему? Потому что. В розетке 220 вольт. Что такое вольт, зачем они в розетке, что с ними делать? Да какая разница. Зато знаю, что в розетку нужно включать телевизор и прочие приборы.
>> No.88999 Reply
>>88981
Ты либо полный идиот, либо тролль. Тебе привели целый ворох способов применения комплексных чисел, однако ты продолжаешь говорить "Купи слона" "Ну это-то все хорошо, а как их применять в реальности?". Напоминает древний сказ о семи красных линиях.
>> No.89000 Reply
>>88997
Науки применяют инженеры-разработчики, а не обыватели, бака!
>>88999
Я и сам их приводил с вами. Но задачи, которые мы решали так и замыкаются на математике и так и не перешли в естественные и другие науки, не говоря о том, чтобы найти применение в технологиях.
>> No.89001 Reply
>>89000
> Науки применяют инженеры-разработчики, а не обыватели, бака!
Сам ты бака.
> Просили применение в этой самой вашей пошлой реальности, например в механике
В реальности даже действительные числа не применяются.
>> No.89002 Reply
>>89000
А разгадка как всегда одна: ты абсолютно невежественен.
Каких примеров ты ждёшь, если твои знания о естественных науках осциллируют в малой окрестности нуля?
  
Про уравнение Шрёдингера слышал?
А то, что теория представлений даёт один из основных инструментов описания элементарных частиц?
А что операторные алгебры -- основной инструмент разработки всевозможных теорий поля?
Конечно, тебе всё это неизвестно.

А вообще самое очевидное применение комплексных чисел в физике я уже привёл тут ==>>88884, но ты этого не понял.
Колебательные процессы, например, переменный ток в электротехнике описывается с помощью комплексных чисел (потому что задание точки бегающей по окружности с начальной фазой равносильно заданию комплексного числа).

В общем, дискуссию пора прекратить, выучи сначала азбуку.
>> No.89003 Reply
>>89002
Да-да. Уравнение Шрёдингера - часть повседневной жизни каждой домохозяйки. А операторные алгебры - такая распространённая вещь, что ими пользуются все россияне каждый день.
>> No.89004 Reply
>>89002
Человек просил конкретную задачу из Ньютоноской физики, то есть без планет и частич. Я лишь хотел, чтобы реквест был хорошо понят.
>>89003
Причём тут домохозяйки, сумасшедший?
>> No.89005 Reply
>>89004
Из всей математики некоторый интерес представляют лишь арифметика и проценты. Действительные числа и комплексные числа бесполезны для обывателя. Такая же ситуация со всеми прочими науками. Механика ньютона нужна только физикам и сдающим ЕГЭ школьникам.

Зачем все россиянцы в обязательном порядке учат матан, физику, химию и прочий астрал?
>> No.89006 Reply
>>88965
Имею ввиду что нибудь легкое и понятное, а то лекции преподавателя полный хаос.
>> No.89007 Reply
>>89004
Ну, я обращался к тому балагуру, который всё никак не найдёт применения комплексным числам.

я надеюсь никто не собирается дискутировать вот с этим ==>>89005 лол
>> No.89008 Reply
>>89005
Комплексных чисел в современных школах вроде и так нету.
>> No.89009 Reply
>>89007
ОК, больше не кормлю.
>> No.89010 Reply
>>89008
Есть же, на математических профилях.
>> No.89011 Reply
>>89010
А при чём тут профили? Об обывателях речь же. С такими вопросами иди лучше в минообр.
>> No.89012 Reply
>>89011
В нынешних школах обязательно есть профили. Во всех школах.
>> No.89013 Reply
>>89012
Офигеть. Но можно с 9-го уйти, так?
>> No.89014 Reply
>>89013
Не, нельзя. Это не по понятиям.
>> No.89036 Reply
>>88965
Попробуй почитать первые одну-две главы "Обыкновенные дифференциальные уравнения" Арнольда

>>88935
Можно лекции посмотреть на эту тему Френкеля, оче простые лекции, не понять невозможно.
YouTube: Mathematics - Multivariable Calculus - Lecture 1
>> No.89078 Reply
Бампану интересной ссылочкой.
> Курт Гедель показал неизбежную неполноту математики: в ней существуют истинные положения, которые невозможно строго доказать. Особое число Ω выявляет еще большую неполноту и свидетельствует о существовании бесконечного множества теорем, которые нельзя вывести из конечного набора аксиом.
http://elementy.ru/lib/430319
>> No.89079 Reply
>>89078
Хоть и доброчан, но как же вы задолбали с этим дерьмом...
>> No.89080 Reply
>>89079
Каждому своё. Можешь скрыть тред, если хочешь.
>> No.89081 Reply
>>89080
Он не говорил о математике, он про эти Гёделе-проблемы, типа много критики и мало конструктива, а также много абстракции мало реальности.
>> No.89085 Reply
>>89081
"Выяснить всегда старайся, что мечом световым разрубить не можешь ты. Важно это жизни твоей для."
Йода Таллисибет Энвандунг-Эстерхази

Математику небесполезно знать, какие вещи можно доказать, а какие принципиально недоказуемы.
>> No.89086 Reply
>>89085
Конкретно эта тема обсуждалась во всех уголках всех борд 100500 раз, это самая обожаемая теорема анонимусов.
>> No.89087 Reply
>>89086
Про теорему Гёделя слышали все, а про в некотором роде её обобщение - не все. Проблема доказательств актуальна и современна. Ей интересуется, например, Стивен Вольфрам, разработчик вольфрамальфы.
>> No.89088 Reply
>>89086
Конечно. Она говорит о достаточно сложных вещах из оснований математики, однако ее кажующаяся наглядность позволяет псевдофилософам ее вставлять в свои разглагольствования, наукоблядки де опять соснули, научный метод бесперспективен, Гедель доказал, а мужики-то и не знали... Чувствую, что сейчас понабегут.
>> No.89089 Reply
>>89079
Хотел бы я кого-нибудь заебать Геделем.
Мимо
>> No.89090 Reply
>>89088
А у меня были знакомые, которые утверждают, что математика сводит с ума и заставляет думать не так, как надо, чтобы добиться успеха и уметь общаться. Они говорят, что логика неверна, а ололо Гёдель это доказывает. Если логика неверна, я тогда вообще не знаю, как мыслить по-другому. Они говорят, я левополушарный и неумею мыслить ололо-нелогически по-другому, поэтому неудачник и не понимаю их. Остановите планету, я сойду.
>> No.89091 Reply
>>89088
Да, вообще любое изложение теорем без строгих формулировок, в бытовых терминах "на пальцах" очень вредно: желающий что-то понять не поймёт ничего, а у дурака наоборот возникнут иллюзии понимания.
>> No.89095 Reply
>>89090
> Если логика неверна, я тогда вообще не знаю, как мыслить по-другому.
Давай определимся, какая логика неверна. Ведь логик много.
Классическая формальная логика основывается на четырёх принципах:
1) принцип тождества - понятия должны оставаться тождественными на протяжении рассуждения
2) принцип противоречия - если суждение утверждает, что объект обладает определённым качеством, а другое суждение утверждает, что объект обладает противоположным качеством, то они не могут быть верны одновременно. Например, не верны одновременно на протяжении одного рассуждения утверждения "Сократ высокий" и "Сократ низкий".
3) принцип исключённого третьего - всегда верно либо суждение, либо его отрицание, третьего не дано. Сократ либо высокий, либо не высокий. Этот принцип существенно отличается от предыдущего отсутствием использования смысла утверждения. То есть в формальной логике качество "высокий" не является отрицанием качества "низкий", это просто некие качества.
4) принцип достаточного основания - любая мысль, чтобы иметь силу, должна быть обоснована.

От каждого из этих принципов можно отказаться. Откажешься от первого, и в рассуждении "что лучше вечного блаженства? ничто. значит, атеисты, обещающие после смерти ничто, предпочтительнее христиан, обещающих после смерти вечное блаженство" не будет ошибки. Откажешься от второго - и твой Сократ сможет быть одновременно и низким, и высоким, и молодым, и старым. Откажешься от третьего - то будешь иметь кота, который и жив, и не жив одновременно. Откажешься от четвёртого, и сможешь ссылаться на высосанные из пальца утверждения.

Есть нечёткая логика. В ней верность утверждения описывается числом из [0;1], то есть закон исключённого третьего замещается иным законом. Например, утверждение "Сократ синий" может быть верным на 0.99876454(9). В формальной логике утверждение "Сократ синий" либо верно, либо не верно.
Есть также математическая логика. Она рассматривает суждения как набор символов, некоторым образом интерпретируемых. Именно с этой логикой работал Гёдель.
>> No.89099 Reply
>>89095
> 2) принцип противоречия
Это можно объяснить проще: в рассуждении не должно быть взаимоисключающих параграфов.
>> No.89106 Reply
>>89095
Все четыре принципа. Но главные нападки на следствия вообще.
>> No.89108 Reply
>>89090
> заставляет думать не так, как надо, чтобы добиться успеха и уметь общаться.
За остальное не скажу, но это 100% так.
57+ММ, отбрыкался от математики 5 лет назад, нюхаю цветочки, пою песенки, трахаюсь, рублю бабло
>> No.89110 Reply
>>89106
Формальная логика (точнее, первые её три принципа) запилены Аристотелем самолично. Люди, которые не могут принять формальную логику, в своём умственном развитии находятся на уровне древних греков. Уже перестали одеваться в шкуры, но ещё не научились мыслить. С ними бесполезно спорить, анон. Они просто не понимают тебя.
>> No.89112 Reply
>>89110
Эти греки учатся со мной на математическом факультете на отделении механика.
>> No.89113 Reply
>>89112
Печально, чо.
>> No.89119 Reply
>>89108
Как математика может заставлять не так как надо мыслить? Как надо мыслить?
>> No.89120 Reply
>>89119
Для начала стоит спросить, кому надо и зачем надо.
>> No.89121 Reply
>>89120
Мне надо. Чтобы эфективно общаться с людьми, производить впечатление, выигрывать в спорах.
>> No.89122 Reply
>>89121
Мне сказали, что я высокомерен. Я спросил, что означает слово высокомерие, надо понять типа. Мне сказали, что я мыслю математически - от определений, что такой поход мне только навредит.
>> No.89123 Reply
>>89122
Ум сугубо математический будет правильно работать, только если ему заранее известны все определения и начала, в противном случае он сбивается с толку и становится невыносим, ибо правильно работает лишь на основе совершенно ясных ему начал.
>> No.89167 Reply
>>89122
Мышление от определений, на основе "формальной логики Аритотеля" или же на основе каких-нибудь прочих костылей - не совсем мышление. В этом плане у большинства математиков, или "математиков", оно жутко ограничено.
>> No.89168 Reply
File: 11261w200zc0.jpg
Jpg, 11.97 KB, 200×250 - Click the image to expand
edit Find source with google Find source with iqdb
11261w200zc0.jpg
>>89167
> оно жутко ограничено
И замкнуто?
>> No.89172 Reply
>>89168
Да, ещё оно очень уж компактно.
>> No.89177 Reply
>>89091
Если я верно воспринимаю твоё понятие строгих формулировок, то обращу твоё внимание на их отсутствиев каком-либо из твоих постов и в литературе, на которую ты ссылаешься. И в лучшем случае там иносказания и сокращения, которые можно интерпретировать как строгие формулировки.
>>89095
> любая мысль, чтобы иметь силу, должна быть обоснована
> любая мысль, чтобы иметь силу
> быть обоснована
Нужно уточнить в терминах выводимости и справедливости. Обоснованность, видимо, означает выводимость, а иметь силу — быть справедливой. По-моему, это категоричная логика, да?
>>89110
Я оспариваю истинность твоего суждения и полагаю, что ты не знаком с формальной системой. Это неплохая игра, но подавляющая часть науки находится вне логического вывода. Шашки могут служить основой для формальной логики, но ничего нельзя сказать об умственном развитии людей в зависимости от приятия ими шашек. Более того, именно те похожи на греков, кто путает игру с наукой. Об этом говорится в посте >>89123: играть в шашки, не зная правил, невозможно, но не будучи создателем мира, владеющим формулами, можно заниматься его познанием.
>>89167
> Мышление от определений, на основе "формальной логики Аритотеля" или же на основе каких-нибудь прочих костылей - не совсем мышление.
Надеюсь, мне удастся выразить твою мысль фразой, в справедливости которой я убеждён: формальное мышление совсем не научное мышление.
>> No.89186 Reply
>>89167
Во! Вот один из этих! Спрашивайте у него свои ответы!
>> No.89231 Reply
File: Безымянный.png
Png, 8.43 KB, 163×91 - Click the image to expand
edit Find source with google Find source with iqdb
Безымянный.png
Вот есть ряд, необходимо исследовать на сх\расх.
Короче я думал решать по первому признаку сравнения, но понял что тупо не могу этот рядс сравнивать с рядом аналогичным, только где n была бы в 3 степени. И понял что просто так не могу откинуть часть с логарифмом и тупо зафигарить по 2 признаку сравнения. Каким тогда быстрым способом можно его сделать?
>> No.89237 Reply
>>89231
Можно сравнить с \sum 1/n^{3/2}.
>> No.89237 Reply
>>89231
Можно сравнить с \sum 1/n^{3/2}.
>> No.89248 Reply
>>89231
Логарифм, тащемта, растёт медленней любой степенной функции. И куб его немногим лучше.
(n-1)^2 < n^2 - 3(ln(n))^3 <= n^2 для любого натурального n. Что делать дальше, надеюсь, понимаешь?
>> No.89250 Reply
File: 12999335066487.jpg
Jpg, 102.47 KB, 1280×1024 - Click the image to expand
edit Find source with google Find source with iqdb
12999335066487.jpg
В математике все основано на аксиомах - которые принимаются на веру. Поэтому там все в принципе не точно.
Также физика основана на моделях, которые тоже неточны. Но пока все на бумаге и в реале сходится всех устраивает.

Ну и вообще математика - только инструмент познания. Поклонение инструменту так же нелепо, как и культ карго.

И кстати манипулировать математиками в пять раз проще, чем простыми работягами. Ибо по жизни в массе как тупые дети, представляющие реальность по книжкам.
>> No.89251 Reply
>>89250
Привет, тролль. Сейчас я буду тебя кормить.
> В математике все основано на аксиомах - которые принимаются на веру.
Нет. Аксиомы - это часть определения. То есть математики не верят в то, что такое-то утверждение истинно, они рассматривают некий абстрактный объект с набором свойств, записанных в аксиомах. Так, теория множеств - это объект с набором свойств, записанных в аксиомах теории множеств.
>> No.89260 Reply
>>89248
> (n-1)^2 < n^2 - 3(ln(n))^3 <= n^2 для любого натурального n
Не для любого, а для n > 256
>> No.89261 Reply
>>89250
> Поклонение инструменту так же нелепо, как и культ карго.
Ага, все математики раз в день идут в храм и поклодяются. Ньютон, акбар.
>> No.89265 Reply
>>89261
И тут физики набигают: "во имя Максвела!"
>> No.89266 Reply
Мне нравится направление треда.
>> No.89271 Reply
>>89250
> В математике все основано на аксиомах - которые принимаются на веру. Поэтому там все в принципе не точно.
Примерно так.
> Ну и вообще математика - только инструмент познания.
Математика — эклектичная исторически сложившаяся куча, части которой имеют различную методологию и в различной степени могут рассматриваться как инструмент познания.
>> No.89274 Reply
Я не пойму как готовиться, математику за первый курс сдал, там все легко училось, ну часть со шпаргалок смотрел, если вылетало.
А вот как сдать ДУ, без понятия. Запоминается только первые 5 вопросов, дальше никак не могу запомнить, все из головы вылетает, есть способ ДУ выучить нормально? И какие чаще всего задают дополнительные вопросы, хочу их выучить по-крайне мере.
>> No.89280 Reply
>>89271
> части которой имеют различную методологию
приведи примеры же
>> No.89281 Reply
>>89274
Ты не вопросы запоминай, ты пойми ДУ.
Пойми начало (осмысли те "кирпичики", которые у тебя есть) и пойми, что ты хочешь получить.
А потом сядь за бумажку и выведи всё.
>> No.89282 Reply
>>89280
Тория множеств из "Числовых систем" Фефермана и ZF.
>> No.89297 Reply
>>89281
Не взлетит понять, всякие леммы Асколи-Арцелла и т.д. вгоняют в полный ступор.
   Или вот, допустим: Сведение задачи Коши для системы дифференциальных уравнений произвольного
порядка к задаче Коши для нормальной системы дифференциальных уравнений первого
порядка. Существование и единственность решений
Что тут от меня хотят я даже не въеду с первого раза.
>> No.89454 Reply
Матемачан, что такое "сечение Додекинда"? Будет достаточно понятия "на пальцах". Нагуглить конкретное определение или пример как-то не удалось.
>> No.89459 Reply
File: oh_you.jpg
Jpg, 43.67 KB, 447×335 - Click the image to expand
edit Find source with google Find source with iqdb
oh_you.jpg
>> No.89469 Reply
>>87526
Анон, посоветуй годный учебник по теории чисел для идиота? Читаю вторую главу "Что такое математика?" Куранта и Робертса, хочу прояснить по хардкору некоторые моменты, ибо в математике разбираюсь OCHE медленно и столь же с трудом.
>> No.89470 Reply
>>89454
Дедекиндово сечение - множество из двух подмножеств A и B. Выполняются два свойства: всякий элемент B больше всякого элемента A, и в A нет максимального элемента. Элементы A и B - рациональные числа.

Каждое действительное число есть дедекондово сечение, то есть двухэлементное множество с указанными свойствами.
>> No.89473 Reply
Господа, а как определяется кросс-вейвлетный спектр от двух дискретных рядов данных X,Y? Подразумевается, что вейвлетное преобразование непрерывное, и, возможно, комплексное. Т.е. ччто-то вроде вейвлета Морле.
>> No.89474 Reply
>>89469
для теории чисел алгебру сначала подучить надо
>> No.89475 Reply
>>89473
Хм. Я не знаю, я слишком студент.
>> No.89496 Reply
>>87526
Анон, а что ты думаешь о программе Вербицкого в частности и вообще о его отношении к сегодняшней математике в России? Я втрокурс одного из лучших вузов страны, раздумываю над тем, чтобы посвятить свою жизнь науке.

http://imperium.lenin.ru/~verbit/MATH/programma.html
>> No.89508 Reply
платиновые темы мат.треда
>> No.89509 Reply
Кто бы мне на вопрос ответил?
>>89473-кун
>> No.89513 Reply
Доброанон, у меня завал.
Через 4 месяца егэ, тем временем я нихуя не знаю.
Готов дрочить матешу с утра и до вечера, а вот с чего начать не знаю. У меня очень много пробелов, половину программы не знаю.
Помогите же, прошу.
>> No.89514 Reply
>> No.89515 Reply
>>89509
Пробовал спрашивать на http://dxdy.ru ?
>> No.89516 Reply
>>89513
Привет, доброанон. Мне надо написать программу. Саму программу написал, но я не могу найти формулы. Мне нужно зная одно из 3ёх, найти остальное неизвестное:
1)радиус шара
2)длина стороны вписанного в шар куба
3)длина стороны описанного около шара куба.
Киньте формулы, а то в интернете, че-то всё вокруг да около, а прямых формул нет
>> No.89517 Reply
>>89516
1)Обозначим радиус шара как r
2)Пусть куб вписан в шар. Тогда диаметр шара d=2r есть диагональ куба. Диагональ куба есть a√3, где a - сторона.
d = a√3
2r = a√3
(2/√3)r = a или r = (√3/2)a
3)диаметр окружности, очевидно, равен стороне описанного куба A.
A=2r или r=A/2
Видно, что сторона вписанного куба a соотносится со стороной описанного куба A так:
A = 2r = √3a
a = (2/√3)r = (2/√3)(A/2) = A/√3

Проверьте, мог ошибиться.
школьник-кун
>> No.89520 Reply
>>89513
Было в прошлых тредах. Четыре месяца мало, но мож и получится. Можешь ещё в ЕГЭ-тредах посмотреть. Я как варианты предложу "Алгебру и математический анализ 10 класс" и продолжение за 11-й Виленкина, Ивашева-Мусатова, Шварцбурда и "Геометрию 7-9" Атанасяна и продолжение за 10-11 классы.
>> No.89524 Reply
File: 1350582614652.png
Png, 964.80 KB, 799×600 - Click the image to expand
edit Find source with google Find source with iqdb
1350582614652.png
>>89517
Спасибо, всё правильно. Вот тебе няшку девушку
>> No.89526 Reply
Щито проходят на первом курсе НМУ и по каким книжкам?
>> No.89527 Reply
>>89526
>>87532, например.
>> No.89530 Reply
>>89520
> Четыре месяца мало, но мож и получится.
Я тут функан за 2 ночи осилил.
Post was modified last time at 2013-01-18 20:37:31
>> No.89534 Reply
Кстати, интересно, остался ли тут кто-нибудь из начинавших ходить в НМУ в этом учебном году?
>> No.89535 Reply
>>89534
Давно хочу тебя спросить: ты учился в НМУ?
>> No.89547 Reply
>>89535
а почему вы спрашиваете?
>> No.89548 Reply
>>89547
А почему вы не отвечаете?
>> No.89550 Reply
Нужен человек, который очень хорошо разбирается в мат. анализе, в частности ряды и векторный анализ. Суть такова. Я решаю задачки, скидываю и показываю, а человек говорит правильно ли я решил и оформил. Все, больше ничего не надо, ничего решать и т.д. делать не надо. Найдутся такие?
>> No.89578 Reply
File: Alg.png
Png, 17.44 KB, 731×361 - Click the image to expand
edit Find source with google Find source with iqdb
Alg.png
А я тут обнаружил, что при переводе некоторых рациональных непериодических дробей, представленных в десятичной системе счисления, в двоичную систему исчисления с помощью пикрилэйтед алгоритма в итоге можно наблюдать наличие какой-то НЕХ: далее будут повторяться чётные числа. И, возможно, без закономерности. Алгоритм зацикливается.
Это свойство выбранного алгоритма перевода или самих систем счисления?
«Думая в другую сторону»: возможно ли существование таких систем счисления, пусть и с бесконечно большими словарями символов, что в них не будут существовать дроби, записываемые не конечным числом цифр?
Может ли случиться так, что иррациональное или трансцендентное число представленное в десятичной СС станет рациональным в, например, гуголплекснойной СС?
Числа сохраняют свою принадлежность ко множествам чисел (Действительные, Комплексные, Натуральные) во всех различных системах счисления, в которых их можно представить?
>> No.89587 Reply
File: achieve.jpg
Jpg, 37.75 KB, 450×324 - Click the image to expand
edit Find source with google Find source with iqdb
achieve.jpg
Вечер в тред, котаны!

Посетила меня тут давеча одна нехитрая мысль: хорошо бы поступить на матфак ВШЭ в магистратуру. Это несложно и, должно быть, интересно. Тоефла у меня нет, и будет только к началу мая, так что за бугор я не успеваю. А в рашке вышка вроде самый ок. Самый простой способ поступить - по олимпиаде, которая будет 23 февраля, то есть через месяц (http://olymp.hse.ru/ma).
Подробней: http://math.hse.ru/magistrates-olimp

Программа олимпиады чуть сложней вступительных испытаний. Еще мне кажется, что программа совпадает с GRE Math Subject Test, поэтому можно и к (по) нему готовиться. В общем, давайте составим 3-недельную программу подготовку и набежим. Дискас.
>> No.89589 Reply
>>89578
свойства математических объектов не зависят от формы записи, что и делает их возможными для изучения
>> No.89594 Reply
>>89587
> В Олимпиаде могут принять участие:
> студенты всех вузов России, стран СНГ и Балтии, завершающие обучение по программам бакалавриата и специалитета
> выпускники бакалавриата и специалитета всех вузов России, стран СНГ и Балтии
А больше, наверное, никто. Так что я не могу. Хотя прикольно было бы.
>> No.89642 Reply
File: tumblr_mggwfwgVDD1s0tnt7o1_400.jpg
Jpg, 41.64 KB, 372×497 - Click the image to expand
edit Find source with google Find source with iqdb
tumblr_mggwfwgVDD1s0tnt7o1_400.jpg
>>89250
А с математиками очень просто - кто-то забухал, кто-то вернулся в академическую теплицу, где они могут хоть как-то доминировать. В реальных проектам им очень тяжело.
>> No.89652 Reply
>>89642
Бухающие математики? Ты что, совсем оухел? И с чего им тяжело в реальных проектах?
>> No.89653 Reply
>>89652
Водовка под картофанчик жеш.
>> No.89702 Reply
>>89652
> И с чего им тяжело в реальных проектах?
Потому что там математика (за исключением элементарной) как правило никому нахрен не вперлась.
>> No.89704 Reply
>>89702
Как тогда математики туда попали? Так это математики, которые по специальности не смогли пробиться, то есть слабые. В стройке мостов всяких, там, производствах усилков, компиляторах, проэктировке акустики помещений и прочих сложных вещах полно математики неэлементарной. Откуда вы все берётесь, думающие, что математика не используется в реальных задачах?>>89702
>> No.89705 Reply
>>89704
Да ещё местные усугубляют мифы, говоря, что они ради астралов этим занимаются или ради вспоминая какиех-то нереалистичных роботов.
>> No.89717 Reply
>>89704
Хорошо вам за бугром. А в рашеньке под реальными праектами как правило понимается интыгриравать САП на краснозалупинском маянезном комбинате, канфигурировать бизнос-процесы, канкулировать РОИ. Такие дела.
>> No.89724 Reply
>>89717
Кто сказал, что в рашеньке есть реальные проекты? А кто сказал, что есть математики?
>> No.89729 Reply
File: 800px-ЩИТО.jpg
Jpg, 84.31 KB, 800×600 - Click the image to expand
edit Find source with google Find source with iqdb
800px-ЩИТО.jpg
File: 800px-ЩИТО.jpg
Jpg, 84.31 KB, 800×600
Your censorship settings forbid this file.
unrated
>>89705
> ради вспоминая какиех-то нереалистичных роботов
>> No.89731 Reply
>>89729
Да так, вспоминая срачи в /b. Представь, что не слышал.
>> No.89732 Reply
>>89731
Думаешь, что в клятой Рашке не делают роботов?
>> No.89733 Reply
>>89732
Мне похуй на роботов. Мы говорили про реальные проэкты. Меня с детство бесило, что я русский, если магнитофон русский - то это хуёвый ммагнитофон, а если магнитофон немецкий или французский, то он охуенен, что а значит русский ни что по сравнению с немцем и французом. Мне похуй на знания. Они иллюзорны. Человек не обязательно идеальный инструмент познания, а знания действуют только в каких-то границам, а математика так и вообще абстрактная. Меня интересует только применение в производстве, чтобы реально создать что-то хорошее. Многие мыслят также, как и я.
>> No.89734 Reply
>>89733
Все науки абстрактны, вообще все. Непосредственно физику и непосредственно химию нельзя применять в производстве, нужен посредник между наукой и производством в виде института.
>> No.89735 Reply
>>89733
> Многие мыслят также, как и я.
И именно поэтому
> если магнитофон русский - то это хуёвый ммагнитофон
"С детство", ага.
>> No.89736 Reply
>>89734
Учёные зачастую интересуются теми, а не другими направлениями в науке исходя из того, что они мнят полезным на производстве. Но так мыслят не все.
>> No.89737 Reply
>>89736
Покажи таких учёных.
>> No.89738 Reply
>>89737
Я не вдавался в персоналии. Вспомню - покажу.
>> No.89739 Reply
>>89736
Не, нужны и те, и другие конечно, я не спорю.
>> No.89740 Reply
>>89738
Ок, очень интересно посмотреть. Я из таких только Лысенко знаю.
>> No.89742 Reply
File: SteamboyInventor.jpg
Jpg, 193.18 KB, 900×504
edit Find source with google Find source with iqdb
SteamboyInventor.jpg
File: cyborg.jpg
Jpg, 88.01 KB, 720×389
edit Find source with google Find source with iqdb
cyborg.jpg

>> No.89743 Reply
>>89742
Очень смешно.
>> No.89933 Reply
File: в1.jpg
Jpg, 3.94 KB, 228×101
edit Find source with google Find source with iqdb
в1.jpg
File: о1.jpg
Jpg, 2.52 KB, 138×58
edit Find source with google Find source with iqdb
о1.jpg

>>89743
Привет, матанач. Дурацкий вопрос: правильно ли я взял интеграл прикриплейтед1?
>> No.89941 Reply
File: N8ecIWT3Hp0.jpg
Jpg, 43.75 KB, 533×293 - Click the image to expand
edit Find source with google Find source with iqdb
N8ecIWT3Hp0.jpg
Суп, матаны. А как примерно выглядит граф перереквизитов для групп Ли?
>> No.89962 Reply
>> No.89975 Reply
>>89737
Миша Вербицкий с его неприятием "игры в бисер" подходит?
>> No.89976 Reply
>>89975
Ну Вербицкий сам по себе крайне эксцентричен, а уж в качестве типичного математика его приводить и вовсе не вариант.
>> No.89993 Reply
>>89941
> граф перереквизитов
Я подумал, что "граф пререквизитов" это некоторая структура, которая сопоставляется группе, типа графа Кэли, и товарищ задумался, как её обощить на группы Ли. Стал гуглить "prerequisites graph" и тут-то до меня дошло о чём речь, лол.
>> No.90009 Reply
File: IMG_20130122_195619.jpg
Jpg, 151.88 KB, 849×517 - Click the image to expand
edit Find source with google Find source with iqdb
IMG_20130122_195619.jpg
Добрач, будь добр, объясни мои ошибки, нужно исследовать сходимость, пытаюсь через Даламбера, фигня получается, может быть я где-то глупую ошибку наделал?
>> No.90013 Reply
>>90009
В 3 строчке знаменатель неправильный ((n+1)+7)^n надо. А вообще думай в сторону экспоненты. У тебя же видоизмененное
(1-7/(n+7))^(n+7) -> (1- 7/z)^z -> ((1 - 1/x)^x)^7 - оче похоже на что-то там, стремящееся к e.
z=n+7;
x=z/7.
>> No.90015 Reply
>>90013
У меня с экспонентой и вторым замечательным пределом проблемы очень сильные.

Ладно, забью на это, следующий вопрос.
ряд (sqrt(n+1))/n^2 как можно решить, может быть к какой нибудь эквиалентности привести или что?
>> No.90041 Reply
>>89975
Нет. Он не думает о производстве, а возражает против гильбертизма-бурбакизма.
>> No.90044 Reply
>>90015
sqrt(n+1) <= sqrt(2n)
>> No.90056 Reply
>>90041
А чем плох Бурбаки?
Мне его работы вообще рекомендовали как лучший старт для чайника.
>> No.90062 Reply
>>90056
Норм Бурбаки, ещё бы упрямо шёл путём логического вывода, вообще бы мне его работы нравились.
>> No.90078 Reply
>>90062
> Бурбаки
> его
От оно как.
>> No.90113 Reply
File: bHjRs.jpg
Jpg, 290.61 KB, 1407×660 - Click the image to expand
edit Find source with google Find source with iqdb
bHjRs.jpg
Матемач, а почему дискретная топология называется дискретной?
>> No.90114 Reply
>>90113
Пока что у меня гипотеза, что если взят какую-нибудь метрику на счетном множестве то топология с этой метрикой всегда будет получаться дискретной.
>> No.90116 Reply
>>90113
> "There's a reason it's simply "theoretical," I can develop theories of my own, doesn't make me a genius.
Поржал, спасибо за пик.
>> No.90117 Reply
>>90114
> всегда будет получаться дискретной
нет
придумайте контрпример
>> No.90132 Reply
File: Бурбаки_французск...
Jpg, 111.13 KB, 1277×546
edit Find source with google Find source with iqdb
Бурбаки_французская_книжка.JPG
File: Бурбаки_русская_к...
Jpg, 89.61 KB, 582×566
edit Find source with google Find source with iqdb
Бурбаки_русская_книжка.JPG

>>90056
> А чем плох Бурбаки?
Во-первых, его трактат - нагромождение ненужных математику вещей. Цель трактата Бурбаки - с помощью явного построения показать, что математика является формальной системой, поэтому трактат Бурбаки имеет не математическую, а метаматематическую ценность. Бурбаки не ставил целью создание понятного введения в математику. Трактат Бурбаки избыточен, если рассматривать его как учебник, и, на мой взгляд, вообще затрудняет понимание математики. Вот, к примеру, есть желание изучать группы. Можно ввести сначала определение множества, затем - магмы, затем - квазигруппы, затем - полугруппы, затем - петли, затем - моноида, затем - группы как петли-моноида; а можно просто ввести определение группы. Нужно изучать группы - так давайте изучать группы, а не ковыряться в основаниях.
Во-вторых, бурбакизмы, десятки их.
В-третьих, Бурбаки встроил свою известную тау прямо в логику и пользовался аксиоматиками, отличными от общепринятых. Например, вместо Цермело-Френкеля использовал свою собственную аксиоматику теории множеств, в которой аксиома выбора - простая теорема.
В-четвёртых, устарел.

>>90078
Да. Его. Намекаете, что Бурбаки работал в соавторстве с кем-то? Полноте вам, оставьте эти подлые инсинуации.
>> No.90134 Reply
>>90132
Если отбросить структуралистскую болтовню, модную в 50х и сосредоточиться на математике, то идеология, предлагаемая бурбакистами, это идеология максимально возможной строгости. И по факту эта идеология стала общепринятой, что сказалось на развитии математики чрезвычайно благотворно.

Трактат Бурбаки это попытка систематизации накопленных на тот момент знаний. Типа атласа конечных групп, он никому не нужен, конечно, но существовать должен. Кроме того, некоторые книжки по математике (а не основаниям), например, по коммутативной алгебре или группам Ли написаны удачно и могут быть использованы в качестве дополнения к основному учебнику или в качестве учебника, если нравится стиль.

Вообще говоря, споры о том как относится к Бурбаки в математическом сообществе это примерно как споры о преимуществах постоянного или переменного тока у инженеров -- дискурс уже не актуален лет сто.
>> No.90137 Reply
>>90132
Ты охуел? Бурбаки - это псевдоним коллектива авторов.
>> No.90138 Reply
>>90137
Это ты коллектив авторов, а Бурбаки - это реальный и весьма уважаемый товарищ из университета Нанкаго, ныне, к сожалению, скончавшийся.
>> No.90153 Reply
>>90138
Чота мне так неудобно стало. Пойду я, пожалуй, повешусь.
>> No.90154 Reply
>>90132
> Цель трактата Бурбаки - с помощью явного построения показать, что математика является формальной системой, поэтому трактат Бурбаки имеет не математическую, а метаматематическую ценность.
Т.е. читать Бурбаки для начала можно с тем же успехом, что и "Основания математики" Рассела?
Печально тогда. Я настолько придурок, что и "Что такое математика" Р.Куранта не могу осилить по-человечески. При виде заданий на соображалку впадаю в ступор (это у меня уже лет пятнадцать такой бзик). Тем не менее плачу, колюсь, но продолжаю попытки врубиться.

Я вообще ради интереса спросил у Вербицкого в его блоге, что бы он посоветовлал дубу, желающему учить алгебру, получил ответ - "Линейная алгебра и аналитическая геометрия" за авторством Кострикина и Манина. А там мне первая страница в клочья разнесла мозг, т.к. я понял, что мне за каждым вторым словом надо лезть в справочники. Я бака.

Можно я буду ИТТ задавать идиотские вопросы и немножко трипфажить?
>> No.90155 Reply
>>90154
> Линейная алгебра и аналитическая геометрия
Линейная алгебра и геометрия

самофикс
>> No.90156 Reply
>>90154
Можно. Расскажи, что ты уже прочитал? Может, что-нибудь посоветую.
>> No.90159 Reply
>>90138
Это коллектив математиков. Какой ещё университет Нанкаго, лол?
ruwiki://Николя_Бурбаки
>> No.90160 Reply
File: facepalm-statue.jpg
Jpg, 26.33 KB, 223×320 - Click the image to expand
edit Find source with google Find source with iqdb
facepalm-statue.jpg
>> No.90168 Reply
>>90160
Омич-наркоман, ты сам-то читал статью по твоей ссылке:
> > Однако такой выдающийся автор не мог скрываться до «бесконечности». Вскоре после войны начали просачиваться сведения, что Бурбаки не одно лицо, а целая группа молодых (в тридцатых годах) французских математиков во главе с Вейлем, Дьёдонне и Картаном. Имена их были впервые упомянуты Андре Деляше в 1949 году.
> The founding members were all connected to the École Normale Supérieure in Paris and included Henri Cartan, Claude Chevalley, Jean Coulomb, Jean Delsarte, Jean Dieudonné, Charles Ehresmann, René de Possel, Szolem Mandelbrojt and André Weil
А ещё Жан-Пьер Серр, с которым вёл тёрки Арнольд. А ты всё веришь в байки.
>> No.90170 Reply
>>90168
Пристрелите его кто-нибудь?
>> No.90182 Reply
>>90170
Ты излишне саркастичен. Настолько саркастичен, что даже я начинаю думать, не упорот ли ты?
>> No.90186 Reply
>>90182
Отстань от меня.
>> No.90187 Reply
File: yo-dawg.jpg
Jpg, 48.72 KB, 551×700 - Click the image to expand
edit Find source with google Find source with iqdb
yo-dawg.jpg
>>90156
По сути я преодолевая панику заставил себя прочитать первые две главы книжки "Что такое математика?" Р. Куранта - про математическую числовую систему, системы счисления и самые основы теории чисел. Беда в том, что я и это не очень-то понимаю. Все остальные накупленные я читал, но до первого непонятного момента, вгонявшего меня в двухдневную паническую атаку. Если я что-то не могу решить, я паникую и теряю остатки соображения, но ведь без решения задач ничего не добьешься.

Чтобы как-то обрисовать ситуацию со своей больной головой, чем, надеюсь, смогу помочь доброанону представить, что можно мне посоветовать, расскажу небольшую кулстори про себя и математику. Суть проста - я всегда боялся математики. Мой отец мог легко стать к.ф-м.н., но не фортануло - пошел работать чтобы прокормить семью. Когда мне было 4 года, он начал спиваться, и в результате все мои домашние работы в школе сводились к тому, что мне сначала долго пытались что-то объяснить, затем я с трудом что-то делал под руководством, потому что боялся что мне в лоб дадут, затем была следующая задача и все повторялось. В алгебре я хоть иногда сам понимал как решать примеры, в геометрии я тупил неимоверно. Школа кончилась, настал институт, факультет информатики, линал, матан и прочие радости жизни. Батю уже подключал только на сессиях, ибо с пьяной мордой заниматься математикой было противно и жутко вдвойне (хотя в лоб дать у него уже шансов не было - отдача замучала бы).

Тут я думал, что окончательно возненавидел математику. На парах меня вызывали к доске, не могли добиться от меня внятного ответа, сажали, изрядно похохмив насчет того, что мне место на филфаке. Правда были две преподавательницы, не давшие мне совсем уж взывть. Преподавательница алгебры поставила мне "хор." за упорство и то, что я тупо переписывал контрольные по 8-9 раз пока не получится. Преподавательница практики математического анализа иногда давала интересные задачки и просто и по-доброму объясняла довольно сложные вещи, но и то, под конец второго курса у меня началась паника просто при подходе к институту, и даже на ее парах я тупил. К третьему курсу я почти независимо от учебы угорел по инязу и лингвистике - и ввиду социофобии, личных причин и кучи всего отчислился, не дотянув до экватора где-то недели.

Но все-таки я решил более-менее самостоятельно заняться математикой, по ряду причин.
1) Во-первых, математические методы в лингвистике таки нужны, что подразумевает знание матстата, матлогики и теорвера.
2) Во-вторых, я хочу понимать физику, а не быть "гуманитарием" ничего сложнее E=mc^2 в жизни не разбиравшего. Конечно, исследовательская работа в тех областях мне не светит, но я не хочу как баран на новые ворота пялиться на ур-ия ньютоновской механики.
3) В-третьих, не оставляю надежды научиться программировать. Желательно, в приложении к лингвистическим увлечениям, что значит, что мне нужно изучить дисмат, линал и матан, ибо численные методы и компьютерная графика требуют (компьютерная графика на самом деле это просто моя маленькая фантазия - когда-нибудь написать свой 3d-квест, суть такова...).

Готов заниматься самостоятельно, выслушать любой совет анона. Переодически паникую, но это не всегда. Вообще для меня изучение математики не только способ глубже проникнуть в научное мировоззрение, но и борьба с одним из самых сильных страхов.
>> No.90192 Reply
Я думал, что хоть один математик выглядит как-то сексуально и маскулинно, а это вообще генерал...
>> No.90194 Reply
File: 220px-David_Hilbert_1886.jpg
Jpg, 14.58 KB, 220×311 - Click the image to expand
edit Find source with google Find source with iqdb
220px-David_Hilbert_1886.jpg
>> No.90197 Reply
File: lobachevskiy_1094645154_tonnel.gif.jpg
Jpg, 22.97 KB, 300×406 - Click the image to expand
edit Find source with google Find source with iqdb
lobachevskiy_1094645154_tonnel.gif.jpg
>>90194
Нет эпично выглядящих математиков, сударь. Нет, и всё.
>> No.90285 Reply
File: Снимок1.JPG
Jpg, 57.59 KB, 801×594 - Click the image to expand
edit Find source with google Find source with iqdb
Снимок1.JPG
>>87526
Привет анон. Мог бы ты посоветовать мне годные задачники с решениями (или подробными примерами) по разным разделам матана? Больше всего интересуют дифуры, ТФКП, фукц. анализ, матфизика и, самое главное, всевозможные ряды.
Заранее спасибо.
>> No.90286 Reply
>>90285
Демидовича уже осилил? (нет, у него там не только вычислительные задачи)
>> No.90287 Reply
>>90286
Если тот, ссылка на который в прошлом треде - это немного не то. Хотя ряды порешаю, спасибо.
>> No.90292 Reply
>>90285
http://rghost.ru/43344307
Тащемта например.
>> No.90293 Reply
>>90292
Это замечательно. Ряды и ТФКП есть. Спасибо.
>> No.90295 Reply
>>90187
http://rghost.ru/43344978
Очень интересная и понятная книга.
>> No.90311 Reply
>>90295
Спасибо, анон!
Действительно интересная и хорошая книга - как только приду домой - засяду за нее.
Похоже, это мой шанс как следует повторить изучить школьную программу и только потом браться за институтские учебники.
>> No.90324 Reply
File: 1358918704364.png
Png, 856.46 KB, 1200×798 - Click the image to expand
edit Find source with google Find source with iqdb
1358918704364.png
Привет, доброматемач.
Я хотел быть программистом и йоба-прикладным математиком. Учусь вот. Но я вижу слащавых хабраблядей, я слышу слова скрам жаба паттерны сисярп юнит-тесты скуэль носкуэль биай проджект менеджмент аналитики тестеры митинги цээсэс кроссбраузерность джанга друпал похапе ЙОБА.жпг. Я вижу, какие мудилы крутятся в индустрии. Все это отдает ужасающей офисной оргией с котами и шреддерами, в которой я не хочу участвовать. А момент для того, чтобы учиться на математика упущен. Прикладнобляди, если такие есть, расскажите, чому все так плохо, что делать теперь?
>> No.90328 Reply
>>90324
1) Потому что ты унылый постмодернист и мизантроп. Возможно у тебя были плохие отношения в семье с родителями. Почему я не могу работать ради денег, если мне правда нужны деньги? Нахуя мне эти ваши yoba-интересные работы и yoba-оригинальные специальности с ничем не подкреплённой детской неприязнью к офисам? Если хочется что-то делать ради интереса, а не ради денег и реальной полезности, то почему за это должны платить? Для таких целей есть хобби. Науку логичнее продвигать в качестве научного работника или ещё какого ресёрчера, а инженер и программист хоть как использует то, что уже придумано. Может и врач сам будет изготавливать таблетки и рентгеновские аппараты?
2) Если задача требующая математики возникает часто, то она её единожды(ну или почти единожды) реализуют в библиотеке
3) А если она возникает редко, то она возникает редко и редко приходится иметь с ней дело.
>> No.90329 Reply
>>90324
Всё зависит от тебя. Хочешь стать образованным - иди в библиотеку и читай книги. Не хочешь - сдавай сессию с двоечки на троечку.
>> No.90330 Reply
>>90324
Кстати, люди с дохуя интересными специальностями, жалующимися, что не могут получать за неё хорошие деньги уже просто безумно доебали. И почему ещё они меня всё время хотят задеть? Почему я не могу делать полезную работу, удовлетворяя мои материальные потребности? Это делает меня тупым? Тупым бы я был, если бы моего ума не хватало, чтобы удовлетворить мои потребности. Иначе что делает умным и тупым?
>> No.90331 Reply
>>90330
Ты не меняешь этот мир, ты живёшь.
>> No.90332 Reply
>>90331
И в какую же сторону мне его менять? Опять таки, самый главный вопрос ты проигнорил! Почему мне должны платить деньги за то, что изменяю мир? Это мне придётся делать за свой счёт!
>> No.90333 Reply
>>90332
> мне должны платить
Хватит считать себя чьей-то вещью, которую кто-то будет обслуживать. Нужны деньги - бери.
> И в какую же сторону мне его менять?
В какую хочешь. Не хочешь? Значит, ты не эпичен.
>> No.90335 Reply
>>90333
> Нужны деньги - бери.
Правильно, конечно. Но только не "бери", а "сделай" деньги. Но делать деньги то всё равно придётся с помощью полезного дела, а не интересного, даже над тобой не будет начальников. А работа на начальника всё равно не делает тебя его вещью. Ты просто продаёшь свой труд, вот и всё.
> Не хочешь? Значит, ты не эпичен.
В этом и состоит суть постмодернистов - оскорблять и взывать только к чувственным оскорбительным словцам, но никакой малейшей попытки что-либо обосновывать. Я не обязан ничего тебе менять. Хочешь менять - бери и меняй, а не сиди тут говно разбрасывай. Ты не меняешь ничего, ты говно разбрасываешь.
>> No.90336 Reply
>>90335
> Ты просто продаёшь свой труд, вот и всё.
Ты просто продаёшь себя, вот и всё.
>> No.90337 Reply
>>90335
Меня не беспокоят деньги, меня просто заебало, что все считают, что у меня отвратительная работа и меня считают за это быдлом. Почему я должен себе проблемы от балды придумывать? Почему мне не может быть интересна архитектура программного обеспечения, сетевые взаимодействия? И кстати интересный проект с большим применением математики ты тоже будешь создавать технологии, которые ты тут говном поливаешь, "жаба паттерны сисярп юнит-тесты", тебе даже похуй на математику, тебе всё лишь бы сделать по-другому. Какая бы работа ни была всё лишь не как у всех. Чем тебе не нравятся C# и Java? А? На них не может быть интересных проектов с математикой?
>> No.90338 Reply
>>90336
Отменим разделение труда? Оно не этично?
>> No.90339 Reply
>>90337
А ты вообще какого чёрта припёрся ко мне на кафедру математики со своими паттернами? Свали отсюда в лиспач и наверни там добра.
>> No.90340 Reply
>>90336
Кстати, сам то как деньги "берёшь"?
>> No.90341 Reply
>>90339
Товарищ доказывает, что паттерны на интересных работах, где требуется математика не нужны. Разве он прав?
>> No.90342 Reply
>>90341
Товарищ оффтопит. Ты тоже оффтопишь. Тут математика, а не кризисы среднего возраста и не паттерны. Ок?
>> No.90343 Reply
>>90342
ОК, бро. Кстати у меня у самого спецальность "прикладная математика", но почему-то даже в моей научной работе патерны применяются.
>> No.90349 Reply
File: yrsaH.jpg
Jpg, 125.52 KB, 638×1024 - Click the image to expand
edit Find source with google Find source with iqdb
yrsaH.jpg
Ничего себе вы здесь наговорили, гадостей-то сколько. А мне казалось, что я совсем невинный вопрос задаю. Извини, программист-кун, я тебя быдлом не считаю, но и деятельностью твоей заниматься не хочу. Просто вот стою на распутье и не знаю, чего делать. Решил на доброчан за советом сходить.
>> No.90351 Reply
>>87526
a=0.(9)
10a=9.(9)
10a=9+a
10a-a=9
a(10-1)=9
9a=9
a=1
0.(9)=1

Что за дела?
>> No.90352 Reply
>>90351
not this shit again.jpg
>> No.90353 Reply
>>90351
Все так и есть
>> No.90357 Reply
>>90351
Это не парадокс. Этот пример иллюстрирует твоё непонимание, что такое действительное число. Объясняю в который уже раз. да, я знаю, без введения фундаментальных последовательностей объяснение неполноценно и я пишу ужасно нестрого, но сойдёт и так

Натуральные числа - это числа 1, 2, 3, 4, 5, ... Сейчас я построю действительные числа из натуральных.
Целые числа - это числа 0, ±1, ±2, ±3, ±4, ±5, ...
Дробь - это двухэлементная пара (n,z) или, другое обозначение, n/z, где n-натуральное, а z - целое.
Две дроби a1/a2 и b1/b2 эквивалентны, если a1b2 = b1a2.

Рациональное число - это множество эквивалентных друг другу дробей.
Например R1 = {1/2; 2/4; 4/8; 8/16; etc} или R2 = {1/1; 2/2; 3/3; 4/4; etc.} R1 и R2 - рациональные числа.
Каждое рациональное число однозначно определяется некоторой несократимой дробью, входящей в его состав. Так, R1 определяется 1/2, а R2 определяется 1. Отождествим рациональные числа с несократимыми дробями.

Последовательность рациональных чисел - это (говоря неформально) бесконечный набор значений Rn = {{R1, R2, R3, R4, ... }} , где про R1 можно сказать, что оно первое, про R2 - что оно второе и т.д. Иными словами, последовательность - это некоторое отображение натуральных чисел на рациональные.

Когда мы говорим, что последовательность (рациональных) чисел Rn эквивалентна некоторой последовательности (рациональных) чисел Ln, это означает, что какое бы малое (рациональное) число e>0 мы не взяли, мы сможем подобрать такое натуральное (принципиально натуральное) число N, что все члены последовательностей Rn и Ln, начиная с эн плюс первого члена будут отличаться друг от друга не более чем на e. То есть для любого e>0 существует такое N, что для всех n>N |r_n - l_n| < e.

Действительное число - это множество эквивалентных друг другу последовательностей рациональных чисел.
Действительное число однозначно определяется любой из последовательностей, входящих в его состав.

0.(9) - это последовательность {{9/10, 99/100, 999/1000, 9999/10000, ... }}
1 - это последовательность {{1/1, 1/1, 1/1, 1/1, 1/1, ...}}
Последовательности 0.(9) и 1 эквивалентны друг другу. В самом деле, для любого числа a=a1/a2 существует десятичная дробь b=b1/10^k, такая, что b<k. Разность между членами последовательностей 0,(9) и 1, начиная с k+1, будет меньше a, это очевидно. Следовательно, 0.(9) эквивалентна 1. Следовательно, они задают одно и то же вещественное число v:
v = {
    {{9/10, 99/100, 999/1000, 9999/10000, ... }},
    {{1/1,  1/1,    1/1,      1/1,        ...}},
    ...
}
или, если вспомнить, что мы отождествляли рациональные числа с дробями и ненадолго отменить это отождествление,
v = {
    {{ {9/10; 18/20; ...  }, {99/100; 198/200; ... }, {999/1000; 1998/2000; ... },   ... }},
    {{ {1/1; 2/2; 3/3; ...}, {1/1; 2/2; 3/3; ... },   {1/1; 2/2; 3/3; ... },         ... }},
    ...
}
Разумеется, в v, помимо 1 и 0.(9), входят и другие последовательности.
Отождествим действительное число с какой-нибудь из последовательностей, входящих в него. Это приводит к удобной записи: мы можем записывать v как 1, или как 0.(9), или какой-нибудь ещё последовательностью, без всей этой тучи скобок.
Примеры действительных чисел (после отождествления с последовательностями): 1; -5; 1/5; pi; e.

Цель достигнута, действительные числа построены с помощью натуральных. Ты понял, что 0.(9) и 1 представляют одно и то же число v (частями которого являются). Это число ради удобства обозначается с помощью только одной из последовательностей. Каждое число равно самому себе, значит v = v, то есть 0.(9) = 1.


P.S. Можно продолжить строить числа дальше. Комплексное число - это пара действительных чисел, кватернион - четвёрка действительных чисел, октава - восьмёрка действительных чисел, седенион - шестнадцать действительных чисел.
>> No.90358 Reply
>>90351
>>90357
Другое объяснение с прологом и эпилогом:
http://yadi.sk/d/Zk0v6s3D0Vz9L (рус., некоторых страниц нет)
http://yadi.sk/d/yc7UsiOT0Vzkl (англ.)
>> No.90360 Reply
>>90357
> b<k
Конечно же, b<a.
>> No.90362 Reply
File: в5.jpg
Jpg, 14.55 KB, 842×241 - Click the image to expand
edit Find source with google Find source with iqdb
в5.jpg
>>87526
Привет, матанач. Я пришёл к тебе с глупым вопросом: на пике интеграл, который у меня получился. Правильно ли я его беру? Если да, то как записать его в маткаде для a=1,t=1?
Т.е. я беру Ф(+00)=0.5, для нижнего предела строю как pnorm(x,-5,2)
Капча "Оказались задания" говорит, что мне нужна помощь.
>> No.90364 Reply
Здравствуй, доброчан. Как жить если из всех математик получается и нравится только дискретная, точнее графы? Еще всякие теории игр, теория принятия решений, теория вероятностей, но это меньше. А ряды, диффуры я просто ненавижу и никогда как следует не получались.
>> No.90365 Reply
>>90364
Как как, нормально жить, просто ты энтрилевелельщик, поэтому пока не можешь в годноту.
>> No.90370 Reply
>>90357
   А если ты придёшь в магазин и вместо 100 копеек дашь 99 копеек, то купишь ли ты товар?

   Прочитал значение слова эквивалентность, синонимы которого «идентичный, одинаковый, равный, такой же». Может я чего не недопонимаю или глупый совсем, но 0.(9)<1 и 0.(9)≠1, зато 1-0.0..1=0.9...9. Объясни ещё разок, доброанон, не дошло.

a=0.(9)
90.(9)=8.9...1, а не 90.(9)=9.
>> No.90371 Reply
>>90370
9x0.(9)=8.9...1, a не 9x0.(9)=9.
>> No.90372 Reply
>>90370
> дашь 99 копеек
Дать можно только 99.(9). Если в хвосте числа будет хоть одна не девятка, то оно будет меньше.
> 0.(9)<1
Нет, здесь равенство.
>> No.90373 Reply
>>90357
Блять! Дошло! Грубо говоря, есть квадрат числа, что sqrt(1)=a, но a^2=0.(9). И там суть такая же.
>> No.90375 Reply
>>90373
В самом деле, бесконечное кол-во цифры 9 после нуля и запятой, эквивалентны 1.
>> No.90381 Reply
>>90370
0.(9) - это действительное число. Оно предназначено не для измерения копеек. Это абстракция более высокого порядка, чем натуральные числа.
>> No.90382 Reply
>>90370
Ты не знаешь, что такое число. В школе тебя этому не научили. Если ты не веришь мне, то возведи 5 в степень √2 и докажи, что я не прав.


Действительное число - это не одна нераздельная сущность, а набор рациональных чисел.
Действительное число выглядит вот так:
{{1,1,1,1,1,1,...}, {0.9, 0.99, 0.999, 0.9999, ... }, {2, 3/2, 4/3, 5/4, 6/5, 7/6, 8/7, ... } и т.п.}
1 - это сокращение для {1,1,1,1,1,1,...}.
0.(9) - это сокращение для {0.9, 0.99, 0.999, 0.9999, ... }
1, 0.(9) и {2, 3/2, 4/3, 5/4, 6/5, 7/6, 8/7, ... } - части одного и того же числа.
Сие число однозначно определяется всякой своей частью.

Ага?
>> No.90384 Reply
>>90362
Почему так?
>> No.90389 Reply
>>90357
>>90375
>>90381
Всё-таки какой низкий уровень "математиков" доброчана. На вики, правда, ненамного лучше.
Даже не знают, что бесконечное повторение 9 в хвосте запрещено и не является альтернативной записью чего бы то ни было / 1.
Ещё хотите с фундаментальными последовательности доказать обратное - не докажете. Всё, что сможете, это апеллировать к отношению эквивалентностей... И сразу за этим я бы советовал прочесть определение оного. Ну, то есть, ваши "пруфы" сводятся к хождению вокруг да около и голой софистике.
>> No.90390 Reply
>>90389
> Даже не знают, что бесконечное повторение 9 в хвосте запрещено
Кем?
>> No.90391 Reply
>>90390
Некоторыми уважаемыми авторами. Другими хорошими авторами подразумевается, как очевидное - именно поэтому вы никогда и нигде не найдёте попыток доказательства равенства одного другому (если речь идёт не о сумме ряда) или просто хотя бы таких утверждений.
Если найдёте - будьте добры автора.
>> No.90392 Reply
>>90391
Имена и фамилии уважаемых авторов, будьте любезны? Вангую Садовничего.
>>87532 - вариант с последовательностями
>>90358 - вариант с сечениями
>> No.90394 Reply
>>90392
Помню Munkres, Topology.
Пробежался по Львовскому, можно конкретную страницу и номер положения / упражнения чего бы то ни было? Где там о 0,(9)=1 или что-либо искажающее понятие эквивалентности и далекоидущие выводы основанные на этом?
>> No.90395 Reply
File: top.pdf
Pdf, 0.32 KB, 595×842 - Click the image to get file
top.pdf
>>90393
Глава 3, построение действительных чисел в ней.
> Munkres, Topology.
Какая страница, какой абзац?
>> No.90396 Reply
>>90389
> Даже не знают, что бесконечное повторение 9 в хвосте запрещено и не является альтернативной записью чего бы то ни было
Мсье слышал о периодических десятичных дробях? Мсье может объяснить, чем 0,(9) хуже 0,(3)?
>> No.90397 Reply
Аноны, есть ли какая-нибудь книга типа "математика для чайников"? А то у меня большие проблемы назрели. Третий курс, инженегр-прогромизд. У меня какие-то численные методы, умф, функан сейчас, а я ничего не понимаю. Как и не понимал матан с дифурами, но кое-как сдавал. А сейчас, кажется, не вытяну, преподы требуют хорошего понимания.
И есть ли тут те, кто начинал в математику с 20 лет?
Я хочу все понять, но не могу. Ах.
>> No.90399 Reply
>>90397
Умеешь в матан? Если нет, то пролистай Демидовича (ссылка в предыдущем треде) или Письменного (http://yadi.sk/d/Q8sMFpvJ0SMtb).
>> No.90400 Reply
>>90399
Чуть-чуть
>> No.90402 Reply
>>90395
> Глава 3, построение действительных чисел в ней.
Там об эквивалентности. И там же определение эквивалентности есть. Ничего о том, что вы утверждаете.
>>90395
Прошу прощения, перепутал. Это было в одной хорошей книжке анализа, как найду, постараюсь скинуть.
>> No.90403 Reply
>>90402
> Ничего о том, что вы утверждаете.
А теперь скажите мне, что я, по-вашему, утверждаю.
>> No.90404 Reply
>>90396
0,(3) было бы равно 1, если бы это была четверичная система счисления. Вы уже доебали. Вы, видимо, ко всем теоремам должны так скептично относится. Алсо, все периодически дроби можно записать как a/b, где a и b - целые. Для 0.(9)=1, 0.(3)=1/3
>> No.90405 Reply
>>90404
Мсье может объяснить, почему
> бесконечное повторение 9 в хвосте запрещено и не является альтернативной записью чего бы то ни было
?
>> No.90406 Reply
>>90382
Я понял в чём смысл... Не стоило так напрягаться.
>> No.90407 Reply
>>90403
> 0.(9) - это 1?
>> No.90408 Reply
>>90405
Это из сборника анекдотов? Или "в другой вселенной"?
>> No.90409 Reply
Всё равно 1 не до конца равно 0.(9)...
>> No.90410 Reply
>>90407
Ровно в том же смысле, в котором 1/2 - это 2/4.
>> No.90411 Reply
>>90408
Если это не твои слова, то зачем ты влез? А если это >>90389 твой пост, то я тебя не понимаю.
>> No.90412 Reply
>>90409
Что такое число?
>> No.90413 Reply
>>90412
Абстрактное понятие, которым может быть выражены количественные отношения.
>> No.90414 Reply
>>90413
Почему 0.(9) - это число?
>> No.90415 Reply
>>90414
Если представить числовую прямую, то отрезок от 0 до 0.(9) — будет количественным отношением.
>> No.90416 Reply
>>90415
А где находится 0.(9) на числовой прямой?
>> No.90417 Reply
>>90416
Ближе всего к 1.
>> No.90418 Reply
>>90415
Особенностью числовой прямой является то, что между двумя различными точками всегда можно поставить третью точку. Если 0.(9) и 1 - это две разные точки, то между ними есть третья точка.
Но мы не можем указать точку, которая бы находилась между 0.(9) и 1.
Значит, 0.(9) и 1 - одна и та же точка.
>> No.90421 Reply
>>90418
Есть такой интервал, который будет иметь вид 1-0.0...1. Разве нет?
>> No.90422 Reply
>>90421
1>x≥0.(9) — точнее полуинтервал.
>> No.90423 Reply
>>90418
Главное не злись, братюнь, мне просто интересно узнать...
>> No.90427 Reply
>>90421
> Разве нет?
Нет.
> 0.0...1
Не является записью числа. Как ты себе это представляешь? Бесконечность нулей, а в конце бесконечности единичка?
>> No.90429 Reply
>>90389
> Даже не знают, что бесконечное повторение 9 в хвосте запрещено и не является альтернативной записью чего бы то ни было
Позволь поинтересоваться, откуда эта инфа? В каком учебнике такое можно увидеть?
>> No.90430 Reply
>>90422
>>90422
Возьмём любую точку из этого интервала. Она будет
лежать между
0.9999999... включительно
и
1.0000000.... не включительно
и иметь вид
0.9999a...
Если a меньше девяти, то точка не входит в интервал, так как левее 0.(9), что противоречит условию.
a не может быть больше девяти, так как нет цифр больше девяти. Значит, a обязательно 9.
Значит, любая точка в этом интервале имеет вид 0.9999... А если все точки интервала одинаковы, то интервал суть одна точка.
Такие дела.
>> No.90431 Reply
>>90416
Прямо на 1.
>>90421
Что такое 0.0...1? Как там может умещаться бесконечное число нулей?
Откуда вы все взялись? Какое у вас образование?
>> No.90432 Reply
>>90429
Кочетков Е.С. Алгебра и элементарные функции, 1972, 352 с
"для удобства исключим из рассмотрения бесконечные десятичные дроби с 9 в периоде"
>> No.90433 Reply
>>90430
> Возьмём любую точку из этого интервала.
Этот интервал пуст. Там нет точек. Точнее его вообще не существует. Ты написал, что 0.(9)<1. Так писать категорически нельзя.
>> No.90434 Reply
>>90432
> для удобства исключим из рассмотрения
для удобства исключим из рассмотрения!!!!!!!!!!!!!
>> No.90435 Reply
>>90433
Не я.
> Так писать категорически нельзя.
Почему?
>> No.90436 Reply
>>90434
Все так делают.
>> No.90437 Reply
>>90435
Потому же почему нельзя писать 5<5. Потому что 0.(9) вовсе не меньше 1.
>> No.90438 Reply
>>90436
Не все. У нас такого не было.
>> No.90439 Reply
>>90436
Даже если все так делают, это не значит, что все должны так делать. Технически так делать можно.
>> No.90440 Reply
File: BOURBAKI.jpg
Jpg, 51.49 KB, 200×328 - Click the image to expand
edit Find source with google Find source with iqdb
BOURBAKI.jpg
>>90437
5<5. Также 5>5. Доказано Николя Бурбаки.
>> No.90441 Reply
>>90440
ЧТО??????
>> No.90442 Reply
File: GenBourbaki_petit.jpg
Jpg, 34.16 KB, 303×450 - Click the image to expand
edit Find source with google Find source with iqdb
GenBourbaki_petit.jpg
>>90441
Каждое число больше самого себя. Также каждое число меньше самого себя. Поэтому, в частности, 0 является положительным числом, так как 0>0. Ровно поэтому же 0 является отрицательным числом.
>> No.90443 Reply
>>90431
Давайте введём новое число, которое будет удолетворять условие 1≠0.(9). Попрошу не переходить на личности.
>> No.90445 Reply
>>90443
Если я правильно тебя понял, то такого числа не существует, согласно теореме о полноте множества вещественных чисел.
>> No.90446 Reply
>>90445
Исходя из этой теоремы получается, что 1=0.(9)?
>> No.90447 Reply
>>90443
И на месте какого же члена приведенного ложного уравнения встанет "новое число"?
>> No.90448 Reply
>>90446
Нет, это получается из определения вещественных чисел.
Теорема говорит, что нет чисел, которые были бы между вещественными.
>> No.90449 Reply
>>90447
Похоже, имелось в виду число, которое будет между 0.(9) и 1.
>> No.90450 Reply
>>90447
1-a=0.(9), где a — трансцендентое число.
>> No.90451 Reply
>>90450
a=0, ничего трансцендентного.
>> No.90452 Reply
>>90451
a=0.0...1.
>> No.90453 Reply
>>90452
Бесконечность, которая кончается?
>> No.90454 Reply
>>90453
Да, противоречие... Видимо, действительно, 1=0.(9).
>> No.90455 Reply
>>90454
Почему тебе так сложно это принять? Почему тебя не смущает, что 5/10 = 10/20?
>> No.90457 Reply
>>90455
Потому что очевидно, что чего-то не хватает. Ведь если бы 0.(9) было единицей, то и хотя бы запись была соотвествующая.
>> No.90458 Reply
>>90457
А как насчёт последовательности {2, 3/2, 4/3, 5/4, 6/5, ...}? Она представляет то же число, что и единица.
>> No.90459 Reply
>>90458
В смысле? Не допёр...
>> No.90461 Reply
>>90459
В прямом смысле. Как молекула есть объединение атомов, так действительное число есть множество последовательностей. Единственное отличие - для идентификации действительного числа достаточно знать только один из составляющих его "атомов", например 0.(9) или 1.
>> No.90462 Reply
>>90461
Есть такой вопрос. Получается, что числа 0.0...1 не существует?
>> No.90463 Reply
>>90462
Не существует. Это нелепица.
>> No.90464 Reply
>>90463
А число 0.0...11?
>> No.90465 Reply
>>90464
Аналогично.
>> No.90466 Reply
File: 4fz.jpg
Jpg, 61.50 KB, 497×700 - Click the image to expand
edit Find source with google Find source with iqdb
4fz.jpg
>>90362
Господа-математики, помогите мне! После 2-го знака равно не правильно. Я сводил к дополнительной функции ошибок, вольфрам считает интеграл так: http://www4b.wolframalpha.com/Calculate/MSP/MSP326401a51dg76bbiea4f000[...]&h=36
Но у преподши там прописана функция Ф. Пробовал переходить от erfc(х) к Ф(х), но у преподши в ответах для a=1,t=1 есть только Ф((х+5)/2). Как быть?
>> No.90467 Reply
>>90465
А число 0.0...1...1?
>> No.90478 Reply
>>90459
Прочитай же:
ruwiki://Квадратный_корень_из_2 раздел "Доказательство иррациональности"
Оказывается что дробей не достаточно для того чтобы "обозначить" любое число. Но мы можем построить последовательность из дробей максимально приближающуюся к этому числу.
например 1 1/2 1/3 1/4 .. 1/n .. стремитcя к 0, таких последовательностей очень много, например
-1 -1/2 -1/3 -1/4 .. -1/n .. тоже стремится к 0 или 1 1/4 1/9 ... 1/n^2 .. или даже 0 0 0 0 ...
Множество таких последовательностей называется вещественным числом. Важный момент, чтобы понять с каким числом мы имеем дело достаточно знать только одну последовательность. Остальные ей эквивалентны. Вот тут это понятие формализовано
http://ru.math.wikia.com/wiki/%D0%9E%D1%82%D0%BD%D0%BE%D1%88%D0%B5%D0%[...]D0%B8
  
Таким вот образом 1 = 0.(9) = {0.9 0.99 0.999 0.9999 ....} ~ {1, 1, 1, 1, ..) ~ {1/2 2/3 3/4 4/5 ...n/n+1 .. } ~ много чему еще

Изложение ни в коем случае не претендует на строгость
>> No.90508 Reply
>>90478
Дык а что такое 0.(9)?
>> No.90511 Reply
>>90508
Действительное число есть по определению целое число со знаком и последовательностью цифр после запятой. Вот эта самая последовательность цифр - это отображение из N в {0;...;9}. В этом смысле десятиченые дроби с конечной дробной частью мнятся как имеющие одни сплошь нули начиная с какой то цифры. То есть 1 - это 1.(0). N - это множество всех натуральных чисел. Все элементы N конечны, поэтому в числе 0,0...001 не чему противопоставить еденицу. Нет никакой бесконечно удалённой цифры.
>> No.90513 Reply
>>90511
> Вот эта самая последовательность цифр - это отображение из N в {0;...;9}
А если рассмотреть отображение из N+{~} в {0;...;9}?
>> No.90515 Reply
>>90513
> Действительное число есть по определению
Что у тебя за образование? Кого рассматривать? Это будет не действительное число. Если бы у бабушки был хуй, она была бы дедушкой. Проваливай уже отсюда лучше.
>> No.90516 Reply
>>90515
> Это будет не действительное число.
Ну да. И что с того? Я же не утверждаю, что это будет действительное число. Я спрашиваю, что это будет и какими свойствами это будет обладать?
> Проваливай уже отсюда лучше.
Добра поешь.
>> No.90520 Reply
>>90508
Это была попытка объяснить почему то что ты интуитивно понимаешь под 0.(9) на самом деле является единицей.
>> No.90521 Reply
>>90508
Это была попытка объяснить почему то что ты интуитивно понимаешь под 0.(9) на самом деле является единицей.
>> No.90522 Reply
>>90521
Я другой анон.
>> No.90576 Reply
>>90446
0.(9)=0.9+0.09+0.009+0.0009+...=0.9/(1-0.1)=1
>> No.90579 Reply
>>90576
0.(9)=0.9+0.09+0.009+0.0009...=0.9/(1-0.7)=3. И что с того?
>> No.90580 Reply
>>90579 — это имеется в виду, что я не понимаю, к чему там делитель...
>> No.90588 Reply
>>90132
> Можно ввести сначала определение множества, затем - магмы, затем - квазигруппы, затем - полугруппы, затем - петли, затем - моноида, затем - группы как петли-моноида; а можно просто ввести определение группы.
А есть такая книжка?
>> No.90591 Reply
>>90588
> ввести сначала определение множества, затем - магмы, затем - квазигруппы, затем - полугруппы, затем - петли, затем - моноида, затем - группы как петли-моноида
Бурбаки, "Основные структуры анализа", "Алгебра" (в 4 томах).
> а можно просто ввести определение группы
"Теорема Абеля в задачах и решениях".
>> No.90594 Reply
>> No.90596 Reply
>>90576
> 0.(9) = 0.9/(1-0.1)
Равенство необоснованно.

>>90579
> 0.(9) = 0.9/(1-0.7)=3
Равенство неверно.
>> No.90609 Reply
File: d8c8b23a2cdb060c07d88df20affa359_bp.jpg
Jpg, 42.66 KB, 424×600 - Click the image to expand
edit Find source with google Find source with iqdb
d8c8b23a2cdb060c07d88df20affa359_bp.jpg
>>90591
Спасибо.


Password:

[ /tv/ /rf/ /vg/ /a/ /b/ /u/ /bo/ /fur/ /to/ /dt/ /cp/ /oe/ /bg/ /ve/ /r/ /mad/ /d/ /mu/ /cr/ /di/ /sw/ /hr/ /wh/ /lor/ /s/ /hau/ /slow/ /gf/ /vn/ /w/ /ma/ /azu/ /wn/ ] [ Main | Settings | Bookmarks | Music Player ]